Patho Supertable: Papilloma Virus Infection

  • Uploaded by: Dianne Galang
  • 0
  • 0
  • January 2021
  • PDF

This document was uploaded by user and they confirmed that they have the permission to share it. If you are author or own the copyright of this book, please report to us by using this DMCA report form. Report DMCA


Overview

Download & View Patho Supertable: Papilloma Virus Infection as PDF for free.

More details

  • Words: 28,346
  • Pages: 47
Loading documents preview...
TOPNOTCH MEDICAL BOARD PREP PATHO SUPERTABLE by KEVIN ELOMINA, MD For inquiries visit www.topnotchboardprep.com.ph or https://www.facebook.com/topnotchmedicalboardprep/ Foreword: Pathology is very broad subject, and it takes discretion to determine which information is essential for a general practitioner or a medical graduate about to take the board exam. I have compiled the most common, most high-yield, and most noteworthy concepts (according to my discretion, experience during boards, and my training as a Pathology resident) in general and systemic pathology; however, this table is NOT to be used solely. In order to practice integration with other disciplines, I have omitted recurring themes in some chapters. It is up to you to maximize the utility of this handout by integrating related concepts across different subjects. Also, I have included the pages of some important tables in Robbins that you should know by heart. Should you need someone to talk to, with regard to this handout, the lecture presentations, Pathology in general, or personal concerns that preclude you from performing optimally during the review, do not hesitate contact me through Facebook or Instagram. I am willing to listen, and offer some advice and counseling, if necessary. To the one who uses this, my prayers are with you. I fervently hope that this would help you get a good grade in Pathology and in the boards. Study smart, and pray hard. God bless. Kevin (Kei) Elomina, MD Top 7, September 2016 PLE J Facebook: Kevin Elomina Instagram: @keielomina

PATHO SUPERTABLE PART I: GENERAL PATHOLOGY CELLULAR RESPONSES TO STRESS AND TOXIC INSULTS: ADAPTATION, INJURY, AND DEATH QUESTION ANSWER Increase in SIZE of cells resulting in Hypertrophy increased size of organ; cellular adaptation of non-dividing cells (i.e. myocardial fibers). Increase in NUMBER of cells; examples: BPH, Hyperplasia papilloma virus infection Reduction in the size of an organ or tissue Atrophy due to decrease in cell size and number; example: Alzheimer’s disease A reversible change in which one Metaplasia differentiated cell type is replaced by another cell type; example: Barrett esophagus The first manifestation of almost all forms of Cellular injury to cells; due to influx of ions (and swelling water) due to failure of energy-dependent ion pumps. A type of reversible injury, characterized by Fatty change appearance of lipid vacuoles in the (Steatosis) cytoplasm; often seen in cells participating in fat metabolism (liver, heart). Type of cell death, which results from a Necrosis pathologic cell injury; undergoes cellular swelling and eventual pyknosis, karyorrhexis, and karyolysis. It is the irreversible condensation of Pyknosis chromatin in the nucleus of a cell undergoing necrosis or apoptosis; there is nuclear shrinkage with increased basophilia. It is the destructive fragmentation of the Karyorrhexis nucleus of a dying cell. It is the complete dissolution of the Karyolysis

chromatin of a dying cell. Type of cell death, which is energydependent, tightly regulated, and associated with normal cellular functions; often physiologic; the cell undergoes shrinkage and fragmentation. Pathway of apoptosis triggered by loss of survival signals, DNA damage and accumulation of misfolded proteins; inhibited by anti-apoptotic members of the Bcl family. Initiator caspase for intrinsic pathway of apoptosis. Pathway of apoptosis responsible for elimination of self-reactive lymphocytes and damage by cytotoxic T lymphocytes; initiated by TNF receptors. Initiator caspases for extrinsic pathway of apoptosis. Executioner caspases. Regulated cell death that results in necrosis (morphologically); caspase-independent; activates RIP1 and RIP3 complexes that lead to increased ROS and decreased mitochondrial ATP production. Type of apoptosis often seen in microbeinfected cells; caspase 1 releases IL-1, and together with caspase 11, causes cell injury. It involves sequestration of cellular organelles into cytoplasmic autophagic vacuoles that fuse with lysosomes and digest enclosed material; used as a survival mechanism under various stress conditions (i.e. nutrient deprivation). Marker for autophagy; involved in formation of autophagosome. A form of tissue necrosis in which the component cells are dead but the basic tissue architecture is preserved; affected tissues take on a firm texture; often seen in infarcts of all solid organs (heart, spleen, kidney) except the brain. Characterized by digestion of dead cells, resulting in transformation of the tissue into a liquid viscous mass; often seen in infections (pus) and in hypoxic death of cells within the CNS. This term is usually applied to a limb, generally the lower leg, that has lost its blood supply and has undergone coagulative necrosis with superimposed liquefactive necrosis involving multiple tissue layers. Friable, white appearance of necrosis; appears as a structureless collection of fragmented or lysed cells and amorphous granular debris enclosed within a distinctive inflammatory border (granuloma); “cheeselike”. Refers to focal areas of fat destruction, typically seen in acute pancreatitis; there is release of activated pancreatic lipases into the substance of the pancreas and the peritoneal cavity; the foci of necrosis contain shadowy outlines of necrotic fat cells with basophilic calcium deposits (saponification), surrounded by an inflammatory reaction. A special form of necrosis usually seen in immune reactions involving blood vessels;

Apoptosis

Mitochondri al/Intrinsic pathway Caspase 9 Death receptor/ Extrinsic pathway Caspase 8, 10 Caspase 3, 6 Necroptosis

Pyroptosis Autophagy

LC3 Coagulative necrosis

Liquefactive necrosis

Gangrenous necrosis

Caseous necrosis

Fat necrosis

Fibrinoid necrosis

TOPNOTCH MEDICAL BOARD PREP PATHO SUPERTABLE Page 1 of 47 For inquiries visit www.topnotchboardprep.com.ph or https://www.facebook.com/topnotchmedicalboardprep/

TOPNOTCH MEDICAL BOARD PREP PATHO SUPERTABLE by KEVIN ELOMINA, MD For inquiries visit www.topnotchboardprep.com.ph or https://www.facebook.com/topnotchmedicalboardprep/ deposits of immune complexes, together with fibrin that have leaked out of vessels, result in a bright pink and amorphous appearance in H&E stains, called "fibrinoid" (fibrin-like) by pathologists. Abnormal calcium deposition occurring in the absence of calcium metabolic derangements; examples: Psammoma bodies in cancers with papillary architecture and meningioma Calcium deposition in normal tissues occurring in the presence of hypercalcemia; example: calcinosis Effect of caloric restriction on longevity.

Dystrophic calcification

vessel wall without increase in vascular permeability. An extravascular fluid with high protein content; its presence implies an increased vascular permeability, triggered by tissue injury and ongoing inflammatory reaction. Steps in leukocyte recruitment.

Metastatic calcification Increased due to decreased IGF-1 signaling Sirtuins

Proteins that are produced in response to food deprivation; with actions that lead to prolonged longevity (anti-apoptotic, antiROS). CHAPTER 2: INFLAMMATION AND REPAIR QUESTION ANSWER INFLAMMATION Response of vascularized tissues Inflammation to infections and damaged tissues that brings cells and molecules of host defense from the circulation to the sites where they are needed, in order to eliminate the offending agents. General steps of inflammatory A. Recruitment of response. leukocytes B. Regulation of response C. Recognition of injurious agent D. Removal of agent E. Resolution Morphologic hallmarks of acute Vasodilation and inflammation. accumulation of leukocytes and fluid in the extravascular tissue Components of acute 1. Dilation of small inflammation. vessels 2. Increased microvascular permeability 3. Emigration of leukocytes and their activation Most notable mediator that Histamine produces vasodilation. Most common mechanism of Endothelial cell increased microvascular contraction permeability. Initial vascular response to Vasoconstriction injury. An ultrafiltrate of blood, which Transudate contains little protein, little or no cellular material, and low specific gravity as a result of osmotic or hydrostatic imbalance across the

The process of leukocyte accumulation at the periphery of blood vessels. Molecules in the endothelium and leukocyte responsible for rolling.

Molecules responsible for adhesion.

The process of migration of the leukocytes through the endothelium. Molecules in the endothelium responsible for transmigration. Process of leukocyte migration toward sites of infection or injury along a chemical gradient. Exogenous chemoattractants.

Exudate

A. Margination B. Rolling C. Adhesion D. Transmigration Margination P and E-selectins on endothelium with Sialyl-Lewis-X on leukocyte; Glycam-1, CD34 on endothelium with L-selectin on leukocyte ICAM-1 on endothelium with CD11/CD18 integrins (aka, LFA-1, Mac-1) on leukocyte; VCAM-1 on endothelium with VLA-4 on leukocyte Transmigration or diapedesis PECAM-1/CD31 Chemotaxis Bacterial products (Nformylmethionine, lipids) IL-8, C5a, LTB4 Cell-derived mediators

Endogenous chemoattractants. Type of inflammatory mediators that are normally sequestered in intracellular granules and can be rapidly secreted by granule exocytosis or are synthesized de novo in response to a stimulus; examples: Histamine, Serotonin, Cytokines and Arachidonic acid derivatives Type of mediators that are Plasma-derived produced mainly in the liver and mediators are present in the circulation as inactive precursors that must be activated by proteolytic cleavages to acquire their biologic properties; examples: Complement, coagulation and kinin systems Vasoactive amine that results in Serotonin vasoconstriction; sources include platelets and neuroendocrine cells. Most abundant complement. C3 Complement that acts as an C3b opsonin. Anaphylatoxins. C3a, C4a, C5a Membrane attack complex. C5b, C6-C9 Anti-inflammatory cytokines for Lipoxin, TGF-β, IL-10 TOPNOTCH MEDICAL BOARD PREP PATHO SUPERTABLE Page 2 of 47 For inquiries visit www.topnotchboardprep.com.ph or https://www.facebook.com/topnotchmedicalboardprep/

TOPNOTCH MEDICAL BOARD PREP PATHO SUPERTABLE by KEVIN ELOMINA, MD For inquiries visit www.topnotchboardprep.com.ph or https://www.facebook.com/topnotchmedicalboardprep/ regulation of the inflammatory response. Inflammatory response of prolonged duration in which inflammation, tissue injury, and attempts at repair coexist, in varying combinations. Two kinds of cells seen in granulomas.



Accumulation of fluid in tissues or body cavities.

Chronic inflammation Four main mechanisms of edema formation. Epithelioid cells (activated macrophages with pink, granular cytoplasm, resembling epithelial cell) and giant cells

REPAIR Type of tissue whose cells can Labile tissues readily regenerate as long as the pool of stem cells is preserved; examples: Bone Marrow, and Vaginal Epithelium Cells of theses tissues are Stable tissues quiescent and have only minimal replicative activity in their normal state; capable of proliferating in response to injury or loss of tissue mass; examples: Smooth muscles, Endothelium, and Liver Parenchyma Cells of these tissues are Permanent tissues considered to be terminally differentiated and nonproliferative in postnatal life; examples: Neurons, Cardiac muscle Type of repair that happens in Regeneration labile and stable tissues; influenced by growth factors. Type of repair that happens in Connective tissue chronic, severe inflammation, deposition and in permanent tissues. Steps in healing by connective A. Angiogenesis tissue deposition. B. Formation of granulation tissue C. Remodeling of the scar Most notable growth factor in VEGF angiogenesis. Most important cytokine for TGF-β synthesis and deposition of connective tissue. The pink, soft, granular tissue Granulation tissue seen beneath the scab of a skin wound; hallmark of tissue healing; appears by 3-5 days. Histologic findings of granulation Proliferation of tissue. fibroblasts and new, thin-walled, delicate capillaries (angiogenesis) in a loose ECM, often with admixed inflammatory cells, mainly macrophages CHAPTER 3: HEMODYNAMIC DISORDERS, THROMBOEMBOLIC DISEASE, AND SHOCK QUESTION ANSWER

General morphologic appearance of edema. It is an active process resulting from augmented blood flow due to arteriolar dilation; affected tissue is redder than normal, because of engorgement with oxygenated blood. It is a passive process resulting from impaired venous return out of a tissue; tissue has a blue-red color due to accumulation of deoxygenated hemoglobin in the affected tissue. Characterized by alveolar capillaries engorged with blood, with associated alveolar septal edema or focal minute intraalveolar hemorrhage. Pulmonary septa are thickened and fibrotic, with hemosiderinladen macrophages in alveolar spaces. The central vein and sinusoids of the liver are distended with blood, with central hepatocyte degeneration; periportal hepatocytes are better oxygenated. The central regions of the hepatic lobules are grossly red-brown, slightly depressed, and are accentuated against the surrounding zones of uncongested tan, sometimes fatty liver (nutmeg liver); presence of centrilobular necrosis with hepatocyte dropout, hemorrhage, and hemosiderin-laden macrophages. Pathologic form of hemostasis. Components of Virchow triad (abnormalities that lead to thrombus formation). It is a major contributor to the development of arterial thrombi. It is a major contributor to the development of venous thrombi. Any alteration of the coagulation pathway that predisposes to thrombosis; can be primary (e.g. Factor V Leiden, Protein C and S deficiency) or secondary (e.g. Cancer, atrial fibrillation, and prolonged immobilization)

Edema (tissues) or effusion (body cavities) A. Increased hydrostatic pressure B. Decreased oncotic pressure C. Increased vascular permeability Lymphatic obstruction Clearing and separation of ECM, and subtle cell swelling Hyperemia

Congestion

Acute pulmonary congestion

Chronic pulmonary congestion Acute hepatic congestion

Chronic hepatic congestion

Thrombosis Endothelial injury, Stasis, Hypercoagulability Turbulence or endothelial injury Stasis Hypercoagulability

TOPNOTCH MEDICAL BOARD PREP PATHO SUPERTABLE Page 3 of 47 For inquiries visit www.topnotchboardprep.com.ph or https://www.facebook.com/topnotchmedicalboardprep/

TOPNOTCH MEDICAL BOARD PREP PATHO SUPERTABLE by KEVIN ELOMINA, MD For inquiries visit www.topnotchboardprep.com.ph or https://www.facebook.com/topnotchmedicalboardprep/ Laminations composed pale platelet and fibrin deposits alternating with darker red cell– rich layers; signifies formation of thrombus in flowing blood; present in antemortem thrombosis. Most common site of arterial thrombosis. Most common site of venous thrombosis.

A detached, intravascular solid, liquid or gaseous mass that is carried by the blood distal to its point of origin. Most common and most dreaded sequelae of deep venous thrombosis. Embolus occluding a bifurcation in the pulmonary tree; associated with sudden death due to acute right-sided heart failure. A venous embolus, which entered the systemic circulation through an interarterial or interventricular defect. Microscopic fat globules found in the circulation after fractures of long bones or after soft-tissue trauma; can lead to pulmonary insufficiency, neurologic symptoms, anemia, and thrombocytopenia. Gas bubbles within the circulation obstructing vascular flow and causes distal ischemic injury; example: Decompression sickness (bends, chokes) Entry of amniotic fluid into the maternal circulation through a tear in the placental membranes and rupture of uterine veins; presence of marked pulmonary edema, diffuse alveolar damage, and presence of squamous cells in the pulmonary circulation shed from fetal skin, lanugo hair, fat, and mucin. Area of ischemic necrosis caused by occlusion of the vascular supply to the affected tissue. Infarcts that tend to occur in loose tissues and in those with dual

Lines of Zahn

Coronary > cerebral > femoral Superficial or deep veins of the leg (Note: thrombi in superficial leg veins rarely embolize, while thrombi in deep leg veins are the most common sources of venous emboli) Embolus

Pulmonary embolism Saddle embolus

Paradoxical embolus

Fat embolism

Air embolism

Amniotic fluid embolism

Infarct

Red infarct

circulations, previously congested tissues, or when flow is reestablished after an infarction (i.e. after angioplasty of obstructed artery); examples: Pulmonary and Intestinal infarcts Infarcts that tend to occur in solid White infarct organs with end-arterial circulations; examples: Myocardial and Splenic infarcts State of systemic tissue Shock hypoperfusion due to reduced cardiac output and/or reduced effective circulating blood volume. This type of shock results from Cardiogenic shock failure of the cardiac pump, which maybe caused by MI, ventricular arrhythmias, cardiac tamponade or outflow obstruction. This type of shock results from Hypovolemic shock loss blood or plasma volume. This type of shock is caused by Septic shock microbial infection, caused by Gram-negative and Gram-positive bacteria and fungi. Type of shock that occurs in the Neurogenic shock setting of an anesthetic accident or spinal cord injury as a result of loss of vascular tone and peripheral pooling of blood. This type of shock represents Anaphylactic shock systemic vasodilation and increased vascular permeability caused by an IgE-mediated hypersensitivity reaction. CHAPTERS 4: GENETIC DISORDERS QUESTION ANSWER GENETIC DISORDERS Mendelian disorders One dominant allele is enough to produce Autosomal phenotype (heterozygous); diseases with dominant mutations in key structural proteins/receptors are usually inherited in this manner; examples: Huntington disease, NF, Myotonic dystrophy, TSC, ADPKD, Familial polyposis coli, HS, vWD, Marfan syndrome, EDS (some types), OI, Achondroplasia, Familial hypercholesterolemia, AIP Two recessive alleles produce phenotype; Autosomal enzyme deficiencies are usually inherited in recessive this manner: examples: CF, PKU, Galactosemia, Homocystinuria, Lysosomal storage diseases, α-1-antitrypsin deficiency, Wilson disease, Hemochromatosis, Glycogen storage diseases, SCA, Thalassemias, CAH, EDS (some types), Alkaptonuria, Neurogenic muscular atrophies, Freidrich ataxia, Spinal muscular atrophy Most common lysosomal storage disorder; Gaucher most common form is chronic, nondisease neuronopathic form (Type I); acute neuronopathic form (Type II) most severe; microscopically, distended phagocytic cells in spleen, liver, BM, LNs, tonsils, thymus and Peyer patches, with fibrillary cytoplasm (“crumpled tissue paper” appearance).

TOPNOTCH MEDICAL BOARD PREP PATHO SUPERTABLE Page 4 of 47 For inquiries visit www.topnotchboardprep.com.ph or https://www.facebook.com/topnotchmedicalboardprep/

TOPNOTCH MEDICAL BOARD PREP PATHO SUPERTABLE by KEVIN ELOMINA, MD For inquiries visit www.topnotchboardprep.com.ph or https://www.facebook.com/topnotchmedicalboardprep/ In this pattern of inheritance, males are usually affected due to hemizygosity of the allele, but females may express phenotype due to random activation of one X chromosome; examples: DMD, Hemophilia A and B, CGD, G6PD deficiency, Agammaglobulinemia, Wiskott-Aldrich syndrome, DI, Lesch-Nyhan syndrome, Fragile X syndrome In this pattern of inheritance, females in general are more affected than males (in any scenario), because one allele in a female is enough to manifest the trait; examples: Alport syndrome and Vitamin D-resistant rickets Chromosomal disorders Most common of the chromosomal disorders; leading cause of MR; most common cause: nondisjunction of Ch21 during meiosis; diagnostic clinical features: flat facial profile, oblique palpebral fissures, and epicanthal folds. Prominent occiput, mental retardation, micrognathia, low-set ears, short neck, overlapping fingers, CHDs, renal malformations, limited hip abduction, and rocker-bottom feet are features of this syndrome. Microcephaly and mental retardation, microphathalmia, polydactyly, cardiac defects, umbilical hernias, renal defects, and rocker-bottom feet are features of this syndrome. Components of Chromosome 22q11.2 deletion syndrome.

Patients with Ch22q11.2 deletion syndrome are at high risk of these disorders. Condition characterized by ≥ 2 X chromosomes and ≥ 1 Y chromosomes; most common karyotype is 47XXY (90%); important cause of reduced spermatogenesis and male infertility; clinical features include: eunuchoid body habitus, testicular atrophy, micropenis, absence of secondary sex characteristics, etc. Complete or partial monosomy of X chromosome; female hypogonadism in phenotypic females; Most common abnormality is lack of entire X chromosome (45XO) (57%); single most important cause of primary amenorrhea; clinical features include: ovarian streaks, absence of secondary sex characteristics, short stature, cystic hygromas of head neck, preductal

CoA, etc. Type of hermaphrodism with presence of both ovarian and testicular tissue.

X-linked recessive disorders

X-linked dominant disorders

Trisomy 21 (Down syndrome)

Trisomy 18 (Edward syndrome)

Trisomy 13 (Patau syndrome) DiGeorge syndrome (CATCH22) and velocardiofaci al syndrome Schizophrenia and bipolar disorders Klinefelter syndrome

Turner syndrome

True hermaphrodis m Pseudoherma phrodism

Type of hermaphrodism characterized by disagreement between phenotypic and gonadal sex; examples: MRKH syndrome and androgenital syndromes Trinucleotide repeat disorders Property of trinucleotide repeat disorders Anticipation wherein the disease worsens with each successive generation. Most common cause of MR following Fragile XTrisomy 21; involves CGG expansion in syndrome FMR1 gene in X-chromosome; most distinctive feature is macro-orchidism. Autosomal dominant neurodegenerative Huntington disorder; CAG expansion on HTT gene on disease Ch4p16.3; relentlessly progressive and uniformly fatal. Diseases featuring maternal inheritance; Mitochondrial usually, fundamental defect is abnormality disorders in the electron transport chain; examples: MELAS, Leber hereditary optic neuropathy (LHON) Disorders of genomic imprinting Deletion in paternally derived Chromosome Prader-Willi 15q11.2q13; clinical features include: syndrome mental retardation, hypotonia, profound hyperphagia, obesity, small hands and feet, and hypogonadism. Deletion in maternally derived Angelman Chromosome 15q12; clinical features syndrome include: mental retardation, ataxic gait, seizure, and inappropriate laughter “happy puppet”. CHAPTER 5: DISEASES OF THE IMMUNE SYSTEM QUESTION ANSWER Gell and Coombs classification of hypersensitivity Results from activation of TH2 CD4+ Type I helper T cells by environmental Hypersensitivity antigens, leading to the production (Immediate) of IgE antibodies, which become attached to mast cells; examples: Anaphylaxis, Allergies, Bronchial asthma Caused by antibodies that bind to Type II fixed tissue and cell antigens and Hypersensitivity promote phagocytosis and (Antibody-mediated) destruction of the coated cells or trigger pathologic inflammation in tissues; examples: Goodpasture syndrome, Autoimmune hemolytic anemia, Immune thrombocytopenia, Pemphigus vulgaris, ANCA vasculitides, Acute rheumatic fever, Graves disease, Myasthenia gravis, Insulin-resistant diabetes, Pernicious anemia, hyperacute and acute humoral rejection Caused by antibodies binding to Type III antigens to form complexes that Hypersensitivity circulate and may deposit in (Immune-complex vascular beds and stimulate mediated) inflammation secondary to

TOPNOTCH MEDICAL BOARD PREP PATHO SUPERTABLE Page 5 of 47 For inquiries visit www.topnotchboardprep.com.ph or https://www.facebook.com/topnotchmedicalboardprep/

TOPNOTCH MEDICAL BOARD PREP PATHO SUPERTABLE by KEVIN ELOMINA, MD For inquiries visit www.topnotchboardprep.com.ph or https://www.facebook.com/topnotchmedicalboardprep/ complement activation; examples: SLE, serum sickness, Arthus reaction, PSAGN, PAN, Reactive arthritis Cell-mediated immune responses in which T lymphocytes cause tissue injury, either by producing cytokines that induce inflammation and activate macrophages, or by directly killing cells; examples: Allergic contact dermatitis, Multiple sclerosis, Tuberculosis, PPD, RA, IBD, Acute cellular and chronic rejection, Type 1 DM and Hashimoto thyroiditis (the last two have Type 2 components as well) Autoimmune diseases 32/F presents to the clinic with symptoms of fatigue, joint pains, and malar rash. CBC revealed anemia and low platelet count. What is the diagnosis? Hallmark of SLE. Mechanisms of organ damage in SLE.

Most common autoantibody in SLE. Most specific autoantibodies for SLE. Autoantibodies associated with congenital heart block in neonatal lupus. Histopathologic findings in SLE with skin involvement.

Type of LE that associated with Hydralazine, INH, Procainamide, DPenicillamine intake; rarely involves kidneys and brain; rarely associated with anti-dsDNA; associated with anti-histone antibodies. Morphology of Lupus nephritides Immune complexes in mesangium only; no alterations detectable by light microscopy. Immune complexes in mesangium with mild to moderate increase in mesangial matrix and cellularity. Lesions visualized in fewer than half of the glomeruli, segmentally or globally distributed within each glomerulus; with cell proliferation, swelling, and infiltration of neutrophils and fibrinoid deposits. Most serious form and most

Type IV Hypersensitivity (Tcellmediated/delayed)

SLE

Production of autoantibodies Type III (most); Type II (opsonization, hematologic manifestations) ANA (98%) anti-Sm (Smith), antidsDNA (anti-dsDNA correlates with disease activity) Anti-Ro (SS-A), AntiLa (SS-B) LM: Liquefactive degeneration of basal layer, edema at DEJ, mononuclear infiltrates around blood vessels and skin appendages IF: Deposition of Ig and complement at DEJ Drug-induced lupus

Lupus nephritis Type I (Minimal mesangial) Lupus nephritis Type II (Mesangial proliferative) Lupus nephritis Type III (Focal)

Lupus nephritis Type

commonly encountered; involvement of half or more glomeruli; with diffuse hypercellularity and wire-looping due to extnsive subendothelial deposits. Widespread thickening of capillary wall due to subepithelial immune complexes. Complete sclerosis of greater than 90% of glomeruli; presents with end-stage renal disease. A clinicopathologic entity with a triad of dry eyes, dry mouth and other manifestations, such as arthritis. Most common and most important autoantibody detected in Sjogren syndrome. Triad of 1. chronic inflammation (autoimmunity); 2. widespread damage to small blood vessels; 3. progressive interstitial and perivascular fibrosis in the skin and multiple organs. Most common autoantibody associated with diffuse cutaneous systemic sclerosis. Common autoantibody associated with limited scleroderma (limited cutaneous systemic sclerosis). Most common antibody associated with scleroderma renal crisis. Most common extracutaneous manifestation of systemic sclerosis.

Most common cause of death in patients with systemic sclerosis. Mixture of the features of SLE, systemic sclerosis, and polymyositis; associated autoantibody: anti-RNP particlecontaining U1 RNP. Transplantation Immunology 48/M patient underwent kidney transplant suddenly developed bloody urine few hours after the procedure. Nephrectomy was done and revealed a cyanotic, mottled, and flaccid kidney and necrotic kidney cortex. There is neutrophilic accumulation in the arterioles, glomeruli, and peritubular capillaries. Glomeruli undergo thrombotic occlusion of capillaries and fibrinoid necrosis in arterial walls. 45/F presented with signs of kidney failure 2 months after her renal transplant. Immunohistochemical staining reveals both CD4+ and CD8+ lymphocytes. Morphologic findings

IV (Diffuse)

Lupus nephritis Type V (Membranous) Lupus nephritis Type VI (Advanced sclerosing) Sjogren syndrome

Anti-Ro (SS-A) and anti-La (SS-B) (90%) Systemic sclerosis

Anti-DNA topoisomerase I (antiScl 70) Anti-centromere antibodies Anti-RNA pol III (up to 50%) Raynaud phenomenon Interstitial lung disease and pulmonary arterial hypertension Mixed connective tissue disease (MCTD)

Hyperacute rejection

Acute cellular (T-cell mediated) rejection

TOPNOTCH MEDICAL BOARD PREP PATHO SUPERTABLE Page 6 of 47 For inquiries visit www.topnotchboardprep.com.ph or https://www.facebook.com/topnotchmedicalboardprep/

TOPNOTCH MEDICAL BOARD PREP PATHO SUPERTABLE by KEVIN ELOMINA, MD For inquiries visit www.topnotchboardprep.com.ph or https://www.facebook.com/topnotchmedicalboardprep/ showed an extensive interstitial mononuclear infiltrate with edema and mild interstitial hemorrhage. 30/M developed oliguria and Acute antibodysubsequent renal failure 3 weeks mediated rejection after an uneventful kidney transplant. Renal findings showed necrotizing vasculitis with endothelial cell necrosis, neutrophilic infiltration, deposition of complement, antibody and fibrin, and thrombosis. 24/F diagnosed with renal failure Chronic rejection underwent renal transplant. Four years later, increasing levels of creatinine was noted, and biopsy showed an interstitial fibrosis and tubular atrophy, glomerulopathy with duplication of basement membrane, and multilayering of peritubular capillary basement membranes. Interstitial mononuclear cell infiltrates, NK cells and plasma cells were also noted. Complication of hematopoietic stem Graft vs. host disease cell transplantation, wherein immunologically competent cells attack the tissues of the immunocompromised host. Primary and secondary immune deficiencies Defects in adaptive immunity A constellation of genetically Severe combined distinct syndromes with common immunodeficiency feature of defects in both humoral (SCID) and cell-mediated immune responses, making affected infants susceptible to severe recurrent infections by bacteria, viruses, fungi, protozoans, and opportunistic infections. Two common patterns of X-linked: common inheritance of SCID, and the gamma chain associated defects. (receptors of ILs, especially IL-7) AR: Adenosine deaminase deficiency Primary immune deficiency X-linked characterized by absent or agammaglobulinemia markedly decreased numbers of B(Bruton disease) cells in the circulation, with depressed serum levels of all classes of immunoglobulin. Cardinal features of Thymic Cardiac defect (TOF), hypoplasia (DiGeorge syndrome). Abnormal facies, Thymic aplasia, Cleft palate, Hypocalcemia, 22q11.2 chromosomal deletion (CATCH 22) Disorder of lymphocyte activation, Hyper IgM syndrome characterized by absence of CD40L (CD145) that results in lack of classswitching with consequent increase in IgM and decrease in IgA, IgG and IgE. An X-linked recessive disease Wiskott-Aldrich

characterized by thrombocytopenia, eczema, and marked vulnerability to recurrent infection ending in early death. Hereditary angioedema is secondary to: Deficiency of the following complement protein increases susceptibility to infections involving pyogenic bacteria. Deficiency of the following complement proteins increases susceptibility to immune complexmediated disease. Deficiency of the following complement proteins increases susceptibility to Neisseria infections. Secondary immunodeficiencies Most common secondary immunodeficiency; caused by HIV infection, characterized by profound immunosuppression that leads to opportunistic infections, secondary neoplasms, and neurologic manifestations; the virus targets CD4+ T cells The hallmark of AIDS:

syndrome

C1 inhibitor deficiency C3

C1q, C2 and C4

C5-C9

Acquired immune deficiency syndrome (AIDS)

Variable loss of T-cell mediated immunity Dendritic cells

Cells initially infected by HIV in sexual transmission. Glycoprotein present in HIV for gp120 attachment. Glycoprotein present in HIV for gp41 fusion. Most common fungal infection in Candidiasis patients with AIDS. Most common type of lymphoma B-cell lymphomas associated with HIV infection. A disorder of protein misfolding, Amyloidosis causing extracellular deposition of pink or red colored deposits stained with Congo red, with apple-green birefringence in polarized light. Most common organ affected in Kidney amyloidosis and also with the most serious involvement. CHAPTER 6: NEOPLASIA Note: Specific cancers will be discussed in their respective systems. QUESTION ANSWER Nomenclature An abnormal mass of tissue the Neoplasm growth of which exceeds and is uncoordinated with that of the normal tissues and persists in the same excessive manner after the TOPNOTCH MEDICAL BOARD PREP PATHO SUPERTABLE Page 7 of 47 For inquiries visit www.topnotchboardprep.com.ph or https://www.facebook.com/topnotchmedicalboardprep/

TOPNOTCH MEDICAL BOARD PREP PATHO SUPERTABLE by KEVIN ELOMINA, MD For inquiries visit www.topnotchboardprep.com.ph or https://www.facebook.com/topnotchmedicalboardprep/ cessation of stimuli, which evoked the change. Localized neoplasms; usually ends with –oma, except lymphoma, seminoma, dysgerminoma, and hepatoma, and melanoma (these are malignant neoplasms). Neoplasms that invade and destroy adjacent tissues. Malignant neoplasms of epithelial origin; usually spread by lymphatic route; example: colorectal adenocarcinoma. Malignant neoplasms of mesenchymal origin; usually spread by hematogenous route; example: uterine leiomyosarcoma. Benign, but disorganized appearance of tissue indigenous to a particular organ; example: PeutzJegher polyp Cytologically and architecturally normal tissue in an ectopic location; example: Ectopic gastric tissue in Meckel diverticulum Characteristics of malignancies Extent to which neoplastic cells resemble their normal forebears morphologically and functionally. Considered a hallmark of malignancy, which literally means "to form backward"; term used to describe cells with little or no differentiation. Disorderly but non-neoplastic proliferation of cells; described as a loss in uniformity of individual cells and in their architectural orientation. Fibrous tissue formation in response to neoplasm. Dysplastic changes, which involve the entire thickness of the epithelium. Development of secondary implants discontinuous with the primary tumor, in remote tissues; more than any other attribute, this identifies a neoplasm as malignant. Next to metastasis, this is the most reliable feature that distinguishes malignant from benign tumors. Cancer epidemiology Top 3 common cancers in children. Top 3 common cancers in males. Top 3 common cancers in females. Top 3 common cancer mortalities in males. Top 3 common cancer mortalities in females.

Hallmarks of cancer. Benign neoplasms

Malignant neoplasms Carcinomas

Sarcomas

Hamartoma

Choristoma

Differentiation Anaplasia

Dysplasia

Desmoplasia Carcinoma in situ Metastasis

Local invasiveness

ALL > CNS tumors > Burkitt lymphoma Prostate > Lung > Colorectal Breast > Lung > Colorectal Lung > Prostate > Colorectal Lung > Breast > Colorectal

Normal cellular genes whose products promote cell proliferation; examples: RAS (most commonly mutated proto-oncogene in human cancers), and ABL (in CML) Mutant or overexpressed versions of proto-oncogenes that function autonomously without a requirement for normal growthpromoting signals. Genes whose products apply brakes to the cell proliferation; loss of function mutations of such genes lead to carcinogenesis. “Governor of the cell cycle”; a tumor suppressor gene that antiproliferative effects by controlling G1-S checkpoint in the cell cycle; mutated in Retinoblastoma; key element in HPV tumorigenesis. "Guardian of the genome"; a tumor suppressor gene that regulates cell cycle progression, DNA repair, cellular senescence, and apoptosis, and the most frequently mutated tumor-suppressor gene in human cancers; mutated in Li-Fraumeni syndrome. Metabolism unique to cancer cells; High glucose uptake and increased conversion of glucose to lactose (fermentation), which provides carbon moieties necessary for biosynthesis and growth of cancer cells. Most common mechanism of evasion of apoptosis used by cancer cells. The limitless replicative potential of cancer cells are due to the following mechanisms: Mechanisms of angiogenesis in cancer cells. Steps in invasion and metastases of cancer cells.

Self-sufficiency in growth signals; insensitivity to growth inhibitory signals; evasion of cell death; limitless replicative potential; development of sustained angiogenesis; and ability to invade and metastasize Proto-oncogenes

Oncogenes

Tumor suppressor genes Rb

p53

Warburg metabolism

Interference in the intrinsic (mitochondrial) pathway of apoptosis Inactivation of senescence signals and reactivation of telomerase VEGF and oncogenes that stimulate synthesis of VEGF (RAS, MYC and MAPK) A. Loosening of cell-cell contacts B. Degradation of ECM components C. Attachment of novel ECM components

Molecular biology of cancer TOPNOTCH MEDICAL BOARD PREP PATHO SUPERTABLE Page 8 of 47 For inquiries visit www.topnotchboardprep.com.ph or https://www.facebook.com/topnotchmedicalboardprep/

TOPNOTCH MEDICAL BOARD PREP PATHO SUPERTABLE by KEVIN ELOMINA, MD For inquiries visit www.topnotchboardprep.com.ph or https://www.facebook.com/topnotchmedicalboardprep/ D. Migration of tumor cells

infection (cirrhosis), and scarring schistosoma (pipestem fibrosis). VIRAL DISEASES Clinical oncology Multinucleated giant cells with Warthin-Finkeldey Term used to describe the degree Tumor grade eosinophilic nuclear and cytoplasmic cells of differentiation based on inclusions, seen in measles. histologic appearance of the Pink to purple intranuclear inclusion Cowdry Type A tumor. bodies, seen in Herpes simplex virus inclusion bodies (HSV) infections. Infection with Varicella-Zoster virus Intraepithelial Term used to describe the degree Tumor stage (VZV) produces this kind of lesion. vesicles of localization/spread of the Large, atypical cells with "Owl's eye" Cytomegalovirus tumor; usual criteria: location and nuclei are seen in this viral infection. (CMV) size of the primary tumor, nodal There are no morphologic hallmarks Epstein-Barr virus status, and presence of distant for this viral infection, but it induces (EBV) metastases; has more prognostic lymphoid cell proliferation value than tumor grade. (peripheral blood lymphocytosis); Progressive loss of body fat and Cancer cachexia associated with nasopharyngeal lean body mass, accompanied by carcinoma, Burkitt lymphoma, and profound weakness, anorexia, and some forms of HL. anemia in cancer patients; main BACTERIAL DISEASES cytokine implicated is TNF. Common Gram-positive and Gram-negative infections Symptom complexes that occur in Paraneoplastic Main difference between More extensive patients with cancer that cannot syndromes staphylococcal and streptococcal tissue destruction in be readily explained by local and infections. staphylococci distant spread of the tumor of by Suppuration with grayish Corynebacterium elaboration of hormones not pseudomembrane formation, is seen diphtheriae indigenous to the tissue of origin in what bacterial infection? Clue: of the tumor; examples: ectopic "Chinese characters" appearance of ACTH production (Cushing organism syndrome) in Small cell lung Gram-positive intracellular bacilli in Listeria cancer; Hypercalcemia in CSF are diagnostic of what bacterial monocytogenes squamous cell carcinoma (due to infection? Clue: diagnosis is PTHrp expression) granulomatosis infantiseptica. Condition secondary to release of Tumor lysis syndrome Suppuration with tissue necrosis and Bacillus anthracis products of dying cancer cells hemorrhagic lesions are diagnostic during chemotherapy; of what bacterial infection? characterized by: hyperkalemia, Gram-negative intracellular Neisseria gonorrheae hyperphosphatemia, diplococci are diagnostic of what hyperuricemia, and hypocalcemia. bacterial infection? Fleur-de-lis pattern of necrotizing Pseudomonas CHAPTER 7: INFECTIOUS DISEASES pneumonia and perivascular aeruginosa QUESTION ANSWER infiltration of organisms producing a GENERAL PATTERN OF RESPONSE TO INFECTIONS perivascular "blue haze"? Usual pattern of inflammation, Suppurative Buboes, pneumonia and neutrophilic Yersinia pestis observed in infections with sepsis are clinical forms of what extracellular Gram-positiv cocci, and bacterial infection? Gram-negative rods (pyogenic Mycobacteria (Tuberculosis is discussed under Lungs) organisms). Myobacterial infection that causes Mycobacterium Usual pattern of inflammation Mononuclear disseminated disease in patients avium complex observed in chronic infections, and with profound immunodeficiency (MAC) acute viral and intracellular bacteria (AIDS and transplant patients); and parasites. histologically, there is no granuloma Usual pattern of inflammation Granulomatous formation; rather, macrophages with observed in tuberculosis, fungal AFB are seen, due to profound loss of infections and schistosome eggs; CMI. response to infectious agents that Form of leprosy associated with Tuberculous are not easily eliminated. asymmetric peripheral nerve (paucibacillary) Usual response to viral infections Cytopathic/ involvement; formation of leprosy that involves cytopathic changes in Cytoproliferative granulomas, and positive lepromin cells (inclusion bodies and skin test, due to an intact cellmultinucleated giant cells) or mediated immunity. proliferation of host cells. Form of leprosy associated with Lepromatous Usual response to clostridial Tissue necrosis symmetric peripheral nerve (multibacillary) infections, E. histolytica, HBV in liver, involvement; formation of lipidleprosy and Herpesviruses in brain. laden macrophages (lepra cells) with Usually a sequela of chronic Chronic globi (AFB), and a negative lepromin inflammation, seen in chronic HBV inflammation and skin test, due to a depressed cellTOPNOTCH MEDICAL BOARD PREP PATHO SUPERTABLE Page 9 of 47 For inquiries visit www.topnotchboardprep.com.ph or https://www.facebook.com/topnotchmedicalboardprep/

TOPNOTCH MEDICAL BOARD PREP PATHO SUPERTABLE by KEVIN ELOMINA, MD For inquiries visit www.topnotchboardprep.com.ph or https://www.facebook.com/topnotchmedicalboardprep/ mediated immunity. Syphilis Lesion in syphilis characterized by central coagulation necrosis, rimmed by palisading macrophages, and fibroblasts, plasma cell-rich infiltrate, and few organisms. Characteristic lesion in syphilitic aortitis. Forms of neurosyphilis. Anaerobic diseases Myonecrosis with gas bubble formation is caused by Clostridium perfringens. What enzyme is responsible for the tissue necrosis? Pseudomembranous colitis, characterized by mucopurulent exudate reminiscent of a volcano, is associated with C. difficile. What are the toxins elaborated by this organism? Chlamydial diseases Most common sexually transmitted bacterial disease in the world. Main difference between gonococcal and chlamydial urethritis.

Syphilitic gumma

Obliterative endarteritis of the vasa vasorum Meningovascular, Paretic, and Tabes dorsalis α-toxin (lecithinase) Toxin A: chemokine Toxin B: cytotoxin

Chlamydia trachomatis Absence of organisms in chlamydial urethritis

Trypanosome related disease Chagas disease transmitted by Triatoma sp. (Trypanosoma cruzi) (reduviid bug), primarily affects skeletal, smooth, and cardiac muscle (heart, esophagus and colon). Note: Helminths will be covered in Microbiology. CHAPTER 8: ENVIRONMENTAL AND NUTRITIONAL PATHOLOGY QUESTION ANSWER ENVIRONMENTAL PATHOLOGY Air pollution Organ mostly affected in air Respiratory (lungs) pollution. Size of particles in soot that are more Fine or ultrafine deleterious to health. particles (<10 µm) Systemic asphyxiant; important CO cause of accidental and suicidal death; non-irritating colorless and odorless gas; also present in cigarette smoke; clinically characterized by cherry-red discoloration of skin and mucous membranes. CO saturation where systemic 20-30% hypoxia occurs. CO saturation where coma and death 60-70% occurs. Heavy metal poisoning Microcytic hypochromic anemia, Lead involvement of CNS (more common in children), and PNS (more common in adults), and skeletal system are manifestations of poisoning with this heavy metal. Primary targets of mercury Kidney and CNS poisoning. Forms of mercury that are more Metallic and organic associated with CNS damage due to their lipophilic properties. “Poison of kings”, “King of poisons”; Arsenic targets are CVS, GIT and CNS; trivalent compounds are usually toxic; and can cause lung, bladder and skin cancers. Heavy metal that is preferentially Cadmium toxic to kidneys; primary targets are lungs and bones; associated with Itai-Itai disease. Occupational exposures Most readily preventable cause of Smoking death in humans. Addictive substance in tobacco Nicotine smoke. Most commonly abused substance. Alcohol Organs systems most affected in CNS, GIT and liver acute alcohol intoxication. Thermal injuries Most common thermal injury; most Thermal burns common causes are fire and scalding. Common causes of death in burn Shock, sepsis and patients. respiratory insufficiency Most common hyperthermia Heat exhaustion syndrome. Electrical injury

Fungal diseases Gray-white, dirty-looking Candidiasis pseudomembrane, with underlying mucosal hyperemia and inflammation, is characteristic of what fungal infection? Infection with this fungus produces Cryptococcus granulomas in immunocompetent neoformans hosts, and soap-bubble lesions in CNS, in immunocompromised hosts. Fungus with septate hyphae, Aspergillus branching at acute angles (40°). Non-septate hyphae, branching at Mucor right angles. PARASITIC DISEASES Causative agent of malaria, diagnosis Plasmodium depends on demonstration of asexual stages WITH hemozoin pigment. Causative agent of babesiosis; Babesia diagnosis depends on demonstration of parasites in peripheral blood WITHOUT hemozoin, with the characteristic Maltese cross pattern. Etiologic agent of a parasitic disease Leishmania sp. transmitted by sandflies; has cutaneous and visceral forms; parasites in macrophages are characteristic. Etiologic agent of a parasitic disease Trypanosoma brucei transmitted by Glossina sp. (Tse-Tse fly); has hemolymphatic and cerebral phases; diagnosis relies on demonstration of trypanosomes on blood smears. TOPNOTCH MEDICAL BOARD PREP PATHO SUPERTABLE Page 10 of 47 For inquiries visit www.topnotchboardprep.com.ph or https://www.facebook.com/topnotchmedicalboardprep/

TOPNOTCH MEDICAL BOARD PREP PATHO SUPERTABLE by KEVIN ELOMINA, MD For inquiries visit www.topnotchboardprep.com.ph or https://www.facebook.com/topnotchmedicalboardprep/ Two forms of injury caused by electrical injury.

Cardiac/neurologic (arrhythmias and cardiac and respiratory failure), and burns

neuropathy. Deficiency of this trace element leads Zinc to a condition, characterized by periorificial distribution of lesions, called acrodermatitis enteropathica. Radiation injury Clinical consequences of ionizing Cell death, CHAPTER 9: DISEASES OF INFANCY AND CHILDHOOD radiation. teratogenesis, and QUESTION ANSWER carcinogenesis These represent primary errors of Malformation NUTRITIONAL DISORDERS morphogenesis due to an intrinsically s Protein-energy malnutrition abnormal developmental process; example: Polydactyly PEM caused by a decrease in both Marasmus protein and calorie intake, and These result from secondary destruction of Disruptions weight falls below 60% of normal for an organ or body region that was previously sex, height and age. Serum albumin normal in development, due to an extrinsic is normal or slightly reduced. disturbance in morphogenesis; example: Amniotic bands PEM, which occurs when protein Kwashiorkor deprivation is relatively greater than These are due to generalized compression Deformations reduction in total calories; of the growing fetus by abnormal associated with generalized edema biomechanical forces; example: Uterine (due to hypoalbuminemia), flaky constraint paint appearance of the skin, This refers to multiple congenital anomalies Sequence alternating bands of pale and dark that result from secondary effects of a single hair, enlarged fatty liver, apathy, localized aberration in organogenesis. The listlessness and loss of appetite; initiating event may be a malformation, there is also mucosal atrophy and deformation or disruption; example: loss of villi in the small bowel Oligohydramnios (Potter) sequence (Making them lactose intolerant Complete absence of an organ or its anlage. Agenesis initially). Incomplete development of an organ. Aplasia Self-induced starvation, resulting in Anorexia nervosa Underdevelopment of an organ. Hypoplasia marked weight loss; has the highest Absence of an opening, usually of a hollow Atresia death rate of any psychiatric visceral organ. disorder. Most common cause of neonatal mortality. Congenital A condition wherein a patient binges Bulimia anomalies on food and then induces vomiting; Most common cause of congenital Unknown (40more common than anorexia malformation in humans. 60%) nervosa. Most common known cause of congenital Multifactorial Deficiency of this vitamin causes Vitamin A malformations in humans. (20-25%) night blindness, xerophthalmia, Most common genetic cause of congenital Chromosomal keratomalacia, Bitot spots, and malformations in humans. aberrations eventually, corneal ulceration. Elements of the TORCH complex. Toxoplasma, Others (Treponema pallidum), Rubella, Cytomegalovir us, Herpesvirus Condition in children as result of Rickets Second most common cause of neonatal Prematurity vitamin D deficiency causing the mortality. following signs and symptoms: Histologically characterized by eosinophilic Respiratory Craniotabes (parietal bones buckle membranes line the respiratory distress inward with pressure), frontal bronchioles, alveolar ducts and random syndrome of bossing of the head, rachitic rosary, alveoli, which contain necrotic epithelial the newborn pigeon breast deformity, lumbar cells, admixed with extravasated plasma (Hyaline lordosis, and bow legs. proteins (Diffuse alveolar damage). membrane Deficieny of this vitamin causes Vitamin C disease) impaired collagen formation, leading Main component of pulmonary surfactant; Dipalmitoylph to gum bleeding, easy bruisability, which is insufficient in hyaline membrane osphatidylcho and impaired wound healing. disease. line (DPPC) Deficiency of this vitamin leads to Vitamin D A complication of hyperoxic therapy in Retinopathy rickets and osteomalacia. patients with RDS, resulting in of Symptoms of cheilosis, stomatitis, Vitamin B2 neovasculatization of retinal vessels, prematurity glossitis, dermatitis, and corneal deficiency leading to blindness. (ROP)/Retrol vascularization results from ental deficiency of this vitamin. fibroplasia Symptoms of cheilosis, glossitis, Vitamin B6 A complication of hyperoxic injury in Bronchopulm dermatitis, and peripheral deficiency patients with RDS, resulting in alveolar onary TOPNOTCH MEDICAL BOARD PREP PATHO SUPERTABLE Page 11 of 47 For inquiries visit www.topnotchboardprep.com.ph or https://www.facebook.com/topnotchmedicalboardprep/

TOPNOTCH MEDICAL BOARD PREP PATHO SUPERTABLE by KEVIN ELOMINA, MD For inquiries visit www.topnotchboardprep.com.ph or https://www.facebook.com/topnotchmedicalboardprep/ hypoplasia or a decrease in the number of dysplasia mature alveoli; morphologically, there is (BPD) large, simplified alveolar structures, and a dysmorphic capillary configuration. This condition occurs more commonly in Necrotizing very-low-birth-weight infants, as a result of enterocolitis intestinal ischemia, bacterial colonization of (NEC) gut and formula milk feeding; histologically, there is presence of submucosal gas bubbles, transmural coagulative necrosis, ulceration, and bacterial colonization. Results from antibody-induced hemolytic Immune disease in the newborn that is caused by hydrops blood group incompatibility between mother and fetus, leading to edema fluid accumulation; examples: ABO and Rh incompatibility Hydrops caused by cardiovascular defects, Nonimmune chromosomal anomalies, and fetal anemia. hydrops Inherited disorder of ion transport that Cystic fibrosis affects fluid secretion in exocrine glands (CF) and in the epithelial lining of the respiratory, gastrointestinal, and reproductive tracts; most common lethal genetic disease affecting Caucasian populations; involved gene is CFTR-1 in Ch7q31.2. Defined as the sudden death of an infant Sudden infant under 1 year of age which remains death unexplained after a thorough investigation; syndrome characteristic findings include: multiple (SIDS) petechiae of the thymus, visceral and parietal pleura and epicardium, congested lungs with vascular engorgement with or without pulmonary edema; there is also hypoplasia of the arcuate nucleus. Tumors arising from neural crest cells; Neuroblastom composed of small, primitive-appearing a (Classically cells with dark nuclei, scant cytoplasm, and described as a poorly defined cell borders growing in solid mass crossing sheets within a finely fibrillar matrix the midline, (neuropil: eosinophilic fibrillary material vs. Wilms that corresponds to neuritic processes of tumor that neuroblasts); associated with Homerdoes not cross Wright pseudorosettes. the midline) Most common primary tumor of the kidney Wilms tumor in children; second most common primary tumor of the kidney overall (next to RCC); presents grossly as a large, solitary, wellcircumscribed mass; microscopically, a combination of blastemal, stromal and epithelial cell types is observed (Triphasic combination); blastemal components are described as sheets of small blue cells with few distinctive features; stromal cells are fibrocytic or myxoid in nature; epithelial cells take the form of abortive tubules or glomeruli. PART II: SYSTEMIC PATHOLOGY

CHAPTER 10: BLOOD VESSELS QUESTION ANSWER Congenital anomalies These are small spherical Developmental/berry dilatations, typically in the circle of aneurysms Willis. These are abnormal, typically small, Arteriovenous direct connections between arteries fistulas and veins that bypass the intervening capillaries. Focal, irregular thickening of the Fibromuscular walls of medium and large dysplasia muscular arteries; segments of the vessel wall are focally thickened by combination of irregular medial and intimal hyperplasia and fibrosis, causing luminal stenosis. Hypertension Consists of a homogenous pink Hyaline hyaline thickening of the walls of arteriolosclerosis arterioles with loss of underlying structural detail and with narrowing of the lumen; major morphologic characteristic in benign nephrosclerosis. Characteristic of malignant Hyperplastic hypertension, associated with artieriolosclerosis "onion-skin" concentric, laminated, thickening of the walls of arterioles with luminal narrowing; these laminations consist of smooth muscle cells and thickened duplicated basement membrane; associated with necrotizing arteriolitis. Atherosclerosis Characterized by intimal lesions Atherosclerosis called atheromas that protrude into vascular lumina. Composed of lipid-filled Fatty streaks macrophages and smooth muscle cells (foam cells) but are not significantly raised and thus do not cause any disturbance in blood flow; can appear as early as 1 year, and present in virtually all children older than 10 years old. Three principal components of an 1. Cells (SM cells, atheromatous plaque. macrophages, T cells) 2. Extracellular matrix (collagen, elastic fibers, proteoglycans); and 3. Intracellular and extracellular lipid Most common site of Abdominal aorta atherosclerosis. Plaque that has a thick fibrous cap, Stable plaque minimal lipid core, and minimal inflammation. Plaque that has a thin fibrous cap, Vulnerable plaque large lipid core, and greater inflammation. Fates of an atheromatous plaque The luminal surface exposes the Rupture, ulceration bloodstream to highly or erosion thrombogenic substances and induces thrombus formation.

TOPNOTCH MEDICAL BOARD PREP PATHO SUPERTABLE Page 12 of 47 For inquiries visit www.topnotchboardprep.com.ph or https://www.facebook.com/topnotchmedicalboardprep/

TOPNOTCH MEDICAL BOARD PREP PATHO SUPERTABLE by KEVIN ELOMINA, MD For inquiries visit www.topnotchboardprep.com.ph or https://www.facebook.com/topnotchmedicalboardprep/ Rupture of the overlying fibrous cap or the thin-walled vessels in the areas of neovascularization, results in: Discharge of debris into the bloodstream, producing microemboli composed of plaque contents. Increased pressure or ischemic atrophy of the underlying media, with loss of elastic tissue, leading to weakness of the vessel wall. Critical stenosis in coronary arteries Aneurysms and Dissection It is a localized abnormal dilation of a blood vessel or heart. Aneurysm that involves all three layers of the arterial wall, or the attenuated wall of the heart. A breach in the vascular wall leading to an extravascular hematoma that freely communicates with the intravascular space. Two most important causes of aortic aneurysms. This disease can more commonly affects men >50 years old. Lesion usually positioned below the renal arteries and above the aortic bifurcation; can be saccular (dilation of one portion) or fusiform (circumferential dilation). Small blood vessels and vasa vasorum show luminal narrowing and obliteration (Obliterative endarteritis) scarring of the vessel wall and a dense surrounding rim of lymphocytes and plasma cells that may extend into the media; characteristic of the tertiary stage of Syphilis. Arises when blood enters the wall of an artery, as a hematoma dissecting between its layers; often, but not always aneurysmal in origin. Most frequent pre-existing histologically detectable lesion in aortic dissection; characterized by elastic tissue fragmentation and separation of the elastic and smooth muscle cell elements of the media by cystic spaces filled with amorphous proteoglycan-rich extracellular matrix. Vasculitides 38/M presents with severe headache and diplopia associated with fever and fatigue. Area along the course of the superficial temporal artery was painful to palpation. Biopsy done revealed intimal thickening of the artery,

Hemorrhage

Atheroembolism

Aneurysm formation

70%

Aneurysm True aneurysm False aneurysm

Atherosclerosis (AAA) and hypertension (Ascending aortic aneurysm) Abdominal aortic aneurysm (AAA)

Syphilitic aortitis

Arterial dissection

Cystic medial degeneration

Giant cell arteritis (Large arteries)

granulomatous inflammation of the internal elastic lamina with infiltrate of T cells and macrophages. The most likely diagnosis is: Patient initially present with Takayasu arteritis fatigue, weight loss and fever and (Large arteries) then reduced blood pressure and weak pulses in the carotids and upper extremities, ocular disturbances and neurologic deficits. The most likely diagnosis is: Note: Both vasculitides affect large arteries and exhibit granulomatous type of inflammation. In fact, they have the same histologic appearance. It depends on the clinical data to differentiate both diseases. This disease presents with rapidly Polyarteritis nodosa accelerating hypertension, (Small to mediumabdominal pain, bloody stool, sized arteries) myalgia and peripheral neuritis; histologically, there is segmental Remember: PAN can transmural necrotizing be renal or visceral, inflammation that spares the but never pulmonary pulmonary circulation with extensive fibrinoid necrosis and temporal heterogeneity of lesions. The most likely diagnosis is: 4/F presented with fever and Kawasaki disease maculopapular rash on her trunk (Medium-sized) and extremities. This is associated with conjunctival erythema, edema of the hands and feet, and cervical lymph node enlargement; on biopsy, there is segmental transmural necrotizing inflammation with less fibrinoid necrosis. The most likely diagnosis is: Note: Both vasculitides that affect medium-sized arteries have the same histologic appearance; only Kawasaki disease has less fibrinoid necrosis. It depends on the clinical data to differentiate both diseases. A necrotizing vasculitis that Microscopic generally affects capillaries, polyangiitis (Small arterioles and venules, with few or arteries) no immune deposits; necrotizing glomerulonephritis and pulmonary Memory device: PAN capillaritis are common; associated in small arteries that with p-ANCA (MPO-ANCA); also are temporally exhibits leukocytoclasia homogenous (lesions (fragmented PMNs in post-capillary are of same age) venules); thus also the term leukocytoclastic angiitis. Eosinophil-rich and granulomatous Churg-Strauss inflammation involving the syndrome (Small respiratory tract and necrotizing arteries) vasculitis affecting small vessels; ssociated with asthma and blood eosinophilia; associated with pANCA (MPO-ANCA). Causes necrotizing granulomatous Wegener vasculitis with fibroblastic granulomatosis proliferation; Involves small vessels most notable in the upper and lower r; can cause crescentic

TOPNOTCH MEDICAL BOARD PREP PATHO SUPERTABLE Page 13 of 47 For inquiries visit www.topnotchboardprep.com.ph or https://www.facebook.com/topnotchmedicalboardprep/

TOPNOTCH MEDICAL BOARD PREP PATHO SUPERTABLE by KEVIN ELOMINA, MD For inquiries visit www.topnotchboardprep.com.ph or https://www.facebook.com/topnotchmedicalboardprep/ glomerulonephritis; associated with c-ANCA (PR3-ANCA). Characterized by sharply segmental Thromboangiitis acute and chronic vasculitis of obliterans (Buerger medium sized and small arteries, disease) predominantly of the extremities; there is acute and chronic Note: Do not confuse inflammation accompanied by with Berger disease luminal thrombosis, containing (IgA nephropathy) microabscesses composed of neutrophils surrounded by granulomatous inflammation; strong relationship with cigarette smoking. Disorders of blood vessel hyperreactivity Results from an exaggerated Raynaud vasoconstriction of digital arteries Phenomenon and arterioles, inducing paroxysmal pallor or cyanosis of the digits of the hands and feet; can be primary or secondary. Disorders of veins and lymphatics Abnormally dilated, tortuous veins Varicose veins produced by prolonged increase in intraluminal pressure and loss of vessel wall support; veins show wall thinning at the points of maximal dilation with smooth muscle hypertrophy and intimal fibrosis; focal thrombosis and venous valve deformities are common; most common sites are superficial veins of the upper and lower leg. Three sites of varices produced in GEJ (Esophageal the presence of portal varices), Rectum hypertension. (Hemorrhoids), Periumbillical veins (Caput medusae) Usually arises in the setting of Superior vena cava bronchogenic carcinoma, syndrome mediastinal lymphoma, and other SOLs (aneurysms); usually presents with dilation of the veins of the head, neck, and arms with cyanosis; respiratory distress occurs with pulmonary vessel involvement. Usually arises in the setting of Inferior vena cava tumors that invade IVC syndrome (hepatocellular and renal), and venous thrombosis; usually present with Lower extremity edema, superficial abdominal vein distention, massive proteinuria with renal vein involvement. Most common cause of GABHS lymphangitis (usually with accompanying lymphadenitis). Note: Venous thrombosis discussed in Hemodynamic Disorders.

Tumors Benign and tumor-like conditions Most common form of vascular ectasia. Radial, often pulsatile arrays of dilated subcutaneous arteries about a central core that blanch with pressure; associated with hyperestrinism (pregnancy and liver cirrhosis). 3/F with a large "port wine stain" on the right side of her face has occasional seizures. Further examination showed mental developmental delay. She may also have: A. More extensive vascular malformations B. Multiple colonic polyps C. Color blindness D. Alpha thalassemia An autosomal dominant disorder caused by mutations in genes that encode components of TGF-β signaling pathway. Presents with telengiectasia that are widely distributed over the skin and mucus membranes. These are bright red to blue lesions, that vary from a few millimeters to several centimeters in diameter; Unencapsulated aggregates of closely packed, thin-walled capillaries, usually blood-filled and lined by flattened endothelium; vessels are separated by scant connective tissue stroma; can also be filled with lymph (lymphangioma) Appears as red-blue, soft, spongy masses 1-2 cm in diameter, which can affect large subcutaneous areas of the face, extremities, and other body regions; mass is sharply defined but not encapsulated, composed of large, cavernous, blood-filled spaces; can also be filled with lymph (lymphangioma). Borderline malignant lesions Common in patients with AIDS, caused by HHV-8, causing skin lesions that progress from patches, to plaques, to nodules; there are usually plump spindle cells with extravasated erythrocytes and hemosiderin-laden magrophages in the lesions Malignant tumors These are malignant endothelial neoplasms, with varying histology from plump, anaplastic but recognizable endothelial cells producing vascular channels to widely undifferentiated tumors having solid, spindle cell appearance and producing nondefinite blood vessels; endothelial cell markers include

Nevus flammeus (birthmark) Spider telangiectasia

A (Diagnosis is Sturge-Weber syndrome or encephalotrigeminal angiomatosis)

Hereditary hemorrhagic telengiectasia/OslerWeber-Rendu disease

Capillary hemangiomas blood filled and lined with flattened enfothelium

Cavernous hemangioma

Kaposi sarcoma

Angiosarcoma

TOPNOTCH MEDICAL BOARD PREP PATHO SUPERTABLE Page 14 of 47 For inquiries visit www.topnotchboardprep.com.ph or https://www.facebook.com/topnotchmedicalboardprep/

TOPNOTCH MEDICAL BOARD PREP PATHO SUPERTABLE by KEVIN ELOMINA, MD For inquiries visit www.topnotchboardprep.com.ph or https://www.facebook.com/topnotchmedicalboardprep/ CD31 and vWF. CHAPTER 11: THE HEART QUESTION ANSWER Heart failure The left ventricle is hypertrophied Left-sided heart and dilated, with secondary left failure atrial dilation; the lungs are heavy and boggy, with perivascular and interstitial transudate, alveolar septal edema, and intra-alveolar edema; hemosiderin-laden macrophages (heart failure cells) are present; most common cause of right-sided heart failure. Isolated right-sided HF occurring in Cor Pulmonale patients with intrinsic lung disease that result in chronic pulmonary hypertension. Long standing severe right-sided Cardiac cirrhosis HF leads to fibrosis of centrilobular areas, creating this condition. Congenital heart diseases Most CHDs arise in this period. 3-8 weeks AOG Most common genetic cause of Trisomy 21 CHDs. Acyanotic CHDs Common radiographic feature of Increased pulmonary acyanotic CHDs. blood flow Feature that will distinguish PDA Prominent aortic from VSD radiographically. knob (because both show LVH/BVH) Chamber that enlarges in ASD. Right ventricle Smooth-walled defect near the Ostium secundum foramen ovale, usually without atrial septal defect associated cardiac abnormalities; comprises 90% of ASDs; regardless of type, most common congenital heart disease in adults. Natural history of ASDs. Spontaneous closure for small to moderate-sized ASDs Incomplete closure of the Ventricular septal ventricular septum leading to leftdefect (VSD) to-right shunting; the right ventricle is hypertrophied and often dilated; diameter of pulmonary artery is increased because of the increased volume by the right ventricle; most common congenital heart disease overall; most common type is perimembranous (90%). Natural history of VSDs. Small defects usually close spontaneously (common: 1st 2 YOL); Vast majority of lesions that close do so before age 4; moderate to large VSDs are less likely to

Main determinant of clinical outcome in isolated VSDs.

5/w infant presents with tachypnea, diaphoresis, and difficulty feeding; a harsh, continuous machinery-like murmur was noted upon auscultation. The most likely diagnosis is: The reason why PDA in preterm infants has a greater chance of spontaenous closure than in term infants. Drug that can maintain patency of ductus arteriosus; vital in patients with duct-dependent lesions. Cyanotic CHDs The most common cause of cyanotic congenital heart disease. Heart is large and "boot-shaped" as a result of right ventricular hypertrophy; components are: PS, RVH, VSD; and aorta overriding the VSD. Main determinant of clinical outcome in TOF patients. It is a discordant connection of the ventricles to their vascular outflow; defect is an abnormal formation of the truncal and aortopulmonary septa; common in infants of diabetic mothers and in males; radiographically, "egg on the side" appearance Absence of direct connection between RA and RV, due to unequal division of the AV canal. Obstructive lesions Characterized by circumferential narrowing of the aortic segment between the left subclavian artery and the ductus arteriosus; DA is usually patent and is the main source of blood to the distal aorta; RV is hypertrophied and dilated; pulmonary trunk is also dilated; clinically presents as lower extremity cyanosis.

Aorta is sharply constricted by a ridge of tissue at or just distal to the nonpatent ligamentum arteriosum; constricted segment is made of smooth muscle and elastic fibers that are continuous with the aortic media, and lined by thickened intima; DA is closed; proximally, the aortic arch and its vessels are

close Size (<5mm: clinically asymptomatic; >10mm: clinically with failure to thrive and repeated infections) Patent ductus arteriosus (PDA)

PDA in term lacks mucoid endothelium and muscular media PGE2

Tetralogy of Fallot (TOF)

Degree of PS Transposition of the Great Arteries (TGA)

Tricuspid atresia

Preductal "infantile" Coarctation of the aorta Postductal "adult" Coarctation of the aorta

TOPNOTCH MEDICAL BOARD PREP PATHO SUPERTABLE Page 15 of 47 For inquiries visit www.topnotchboardprep.com.ph or https://www.facebook.com/topnotchmedicalboardprep/

TOPNOTCH MEDICAL BOARD PREP PATHO SUPERTABLE by KEVIN ELOMINA, MD For inquiries visit www.topnotchboardprep.com.ph or https://www.facebook.com/topnotchmedicalboardprep/ dilated; LV is hypertrophic; there is upper extremity hypertension, due to poor perfusion of the kidneys, but weak pulses and low blood pressure in the lower extremities; claudication and coldness of the lower extremities also present; enlarged intercostal and internal mammary arteries due to collateral circulation, seen as rib "notching" on X-ray. Ischemic heart disease A condition wherein ischemia causes pain but is insufficient to lead to death of myocardium; can be stable (secondary to increased demand), unstable (crescendo; secondary to decreased supply), or Prinzmetal (secondary to coronary vasospasm). A condition wherein the severity or duration of ischemia is enough to cause cardiac muscle death. Pattern of infarction caused by occlusion of an epicardial vessel, through a combination of chronic atherosclerosis and acute thrombosis; also referred to as STEMI or Q wave MI; infarct involving ≥50% of the myocardial wall thickness. Pattern of infarction caused by plaque disruption or hypotension leading to transient decreases in oxygen delivery, causing circumferential myocardial damage; also referred to as NSTEMI or non-Q wave MI. Heart dominance is determined by:

Cause of acute MR post MI.

This refers to progressive cardiac decompensation (heart failure) following myocardial infaction. Typical morphology of chronic IHD.

Angina pectoris

Acute myocardial infarction (AMI) Transmural infarctions

Subendocardial infarction

Vessel that gives rise to the posterior IV (descending) branch; either RCA (right dominant) or LCX (left dominant); most common: RCA (90%) Most common vessel involved in Left anterior AMI: descending artery (LAD) (40-50%) Note: Table 10-3 in Robbins Basic Pathology, 9th ed. p. 380 or Table 12-5 in Robbins and Cotran Pathologic Basis of Disease, 8th ed. p. 550, or Table 12-5 in Robbins and Cotran Pathologic Basis of Disease, 9th ed. p. 544, is a high-yield table. Make sure you know this by heart. Stain used for infarcts more ~2-3 Triphenyltetrazolium hours old for visualization. chloride Most sensitive and specific cardiac Cardiac troponins biomarkers: (Troponin I and T) Injury to infarcts mediated in part Reperfusion injury by oxygen free radicals generated by increased number of infiltrating leukocytes facilitated by reperfusion; CK-MB is usually used as cardiac biomarker for diagnosis. Consequence of MI Most common site of myocardial Ventricular free wall rupture. (clinically, cardiac tamponade) Fibrinous pericarditis post-MI. Dressler syndrome

Papillary muscle dysfunction (more common); papillary muscle rupture Chronic ischemic heart disease Left ventricular dilation and hypertrophy (Gross) Myocardial hypertrophy, diffuse subendocardial vacuolization and fibrosis (Microscopic) Sudden cardiac death (SCD)

This can result from a lethal arrhythmia following myocardial ischemia; most common cause is CAD (80%); most common mechanism is a lethal arrhythmia (asystole or ventricular fibrillation). Hypertensive heart disease Typical morphology of systemic Left ventricular hypertensive heart disease. hypertrophy Earliest change seen in left-sided Increased transverse HHD. diameter of myocytes Typical morphology of acute cor Right ventricular pulmonale (pulmonary dilation hypertensive heart disease). Typical morphology of chronic cor Right ventricular pulmonale (pulmonary (and often right hypertensive heart disease). atrial) hypertrophy Valvular heart disease The hallmark of this disease is Calcific aortic heaped-up calcified masses on the degeneration/calcific outflow side of the cusps, which aortic stenosis (most protrude into the sinuses of common VHD) Valsalva and mechanically impede valve opening; cusps may become secondarily fibrosed and thickened; most common cause of aortic stenosis. This is characterized by ballooning Myxomatous or hooding of the mitral leaflets; degeneration of the affected leaflets are enlarged, mitral valve redundant, thick and rubbery; the tendinous cords are elongated, thinned and occasionally ruptured; histologically, there is thinning of the fibrosa layer of the valve, accompanied by expansion of the middle spongiosa layer with increased deposition of mucoid material; usual finding in MVP. Rheumatic fever and Rheumatic heart disease Rationale behind autoimmunity in Molecular mimicry rheumatic fever. (Streptococcal M proteins and cardiac self-antigens) Gell and Coombs classification of Type II ARF. Required criteria in diagnosing Evidence of previous rheumatic fever. GABHS infection (ASO in pharyngitis, and anti DNAse B in pyoderma) Jones major criteria in diagnosing Carditis

TOPNOTCH MEDICAL BOARD PREP PATHO SUPERTABLE Page 16 of 47 For inquiries visit www.topnotchboardprep.com.ph or https://www.facebook.com/topnotchmedicalboardprep/

TOPNOTCH MEDICAL BOARD PREP PATHO SUPERTABLE by KEVIN ELOMINA, MD For inquiries visit www.topnotchboardprep.com.ph or https://www.facebook.com/topnotchmedicalboardprep/ rheumatic fever.

Jones minor criteria in diagnosing rheumatic fever. Pathognomonic sign for rheumatic fever; myocardial inflammatory lesions composed of plump activated macrophages (Anitschkow cells), plasma cells, and lymphocytes; can be seen in all three layers of the heart in ARF (pancarditis); rarely seen in chronic RHD (due to replacement by fibrous tissue). Characterized by organization and subsequent scarring, as a sequelae of rheumatic fever; mitral valve most commonly involved with leaflet thickening, commissural fission and shortening, thickening and fusion of the chordae tendinae; fibrous bridging across valvular commissures create "fish mouth" or " buttonhole" deformity Serious infection characterized by microbial invasion of heart valves or mural endocardium, often with destruction of the underlying cardiac tissues; the heart valves are invaded by friable, bulky and potentially destructive lesions called vegetations that are usually in the form of large, irregular masses on the valve cusps that may extend into chordae. Endocarditis of previously normal valves, the most common causative agent is S. aureus. Endocarditis affecting previously damaged or abnormal valves, commonly caused by viridans streptococci. Clinical criteria used in diagnosing infective endocarditis. Characterized by deposition of sterile, non-inflammatory, nondestructive and small (1mm) masses of fibrin, platelets and other blood components on cardiac valves, along the line of closure of leaflets or cusps. Sterile vegetations that can develop on the valves of patients with SLE; small, granular, pinkish vegetations 1-4mm in diameter and can be located on the undersurface of AV valves, on the cords or

Migratory polyarthritis Subcutaneous nodules Erythema marginatum Syndenham chorea Fever Arthralgia Elevated acute phase reactants (e.g. CRP) Aschoff bodies

Chronic Rheumatic Heart Disease

Infective endocarditis (IE)

Acute bacterial endocarditis Subacute bacterial endocarditis Duke criteria Nonbacterial thrombotic endocarditis (NBTE)

Libman-Sacks endocarditis (LSE)

endocardium. The lesions of this disease are distinctive, glistening white intimal thickenings on the endocardial surfaces of the cardiac chambers and valve leaflets; lesions are composed of smooth muscle cells and sparse collagen fibers embedded in an acid mucopolysaccharide-rich matrix; usually right-sided (causing tricuspid insufficiency and pulmonic stenosis).

Cardiomyopathies Cardiomyopathy characterized by progressive cardiac dilation and systolic dysfunction; heart is characteristically enlarged and flabby, with dilation of all chambers; the ventricular thickness may be less than, equal to or greater than normal; characterized by systolic dysfunction; implicated agents and diseases include: alcohol, myocarditis, doxorubicin, and hemochromatosis. Most common mutated gene in cases of DCM. Type of DCM secondary to an autosomal dominant genetic defect on desmosomes that result in right ventricular failure with arrhythmia and sudden death. Characterized by myocardial hypertrophy, abnormal diastolic filling and ventricular outflow obstruction; the heart is thickwalled, heavy and hypercontracting; there is an asymmetrical septal hypertrophy described as "banana-like"; histologically, there is severe myocyte hypertrophy and disarray with interstitial fibrosis; a common cause of sudden death in young athletes. Most common mutated gene in cases of HCM. The ventricles are of approximately normal size or slightly enlarged, the cavities not dilated, and the myocardium is firm; biatrial dilation is common; microscopically, there is interstitial fibrosis, varying from minimal to patchy to extensive and diffuse; characterized by diastolic dysfunction. Myocarditis Most common cause of myocarditis.

Carcinoid heart disease Dilated cardiomyopathy

Titin (20%) Arrhythmogenic right ventricular cardiomyopathy (ARVC) Hypertrophic cardiomyopathy

β-myosin heavy chain Restrictive cardiomyopathy

Viral (Coxsackie A, B,

TOPNOTCH MEDICAL BOARD PREP PATHO SUPERTABLE Page 17 of 47 For inquiries visit www.topnotchboardprep.com.ph or https://www.facebook.com/topnotchmedicalboardprep/

TOPNOTCH MEDICAL BOARD PREP PATHO SUPERTABLE by KEVIN ELOMINA, MD For inquiries visit www.topnotchboardprep.com.ph or https://www.facebook.com/topnotchmedicalboardprep/ Parasitic etiologies of myocarditis.

Immune causes of myocarditis. Pericardial diseases Type of pericarditis found in patients with uremia or viral infection; the exudate imparts an irregular appearance to the pericardial surface (bread and butter pericarditis). Bacterial pericarditis manifests with this type of exudate. Heart is completely encased by dense fibrosis that it cannot expand normally during diastole; sequela of chronic pericarditis. Abnormal fluid accumulation in the pericardial sac; can be serous (from CHF or hypoalbuminemia); serosanguinous (from blunt chest trauma, malignancy, ruptured MI, or aortic dissection), or chylous (from mediastinal lymph node obstruction).

enteroviruses) T. cruzi (Chagas disease), Trichinella spiralis (most common helminthic) Hypersensitivity, giant-cell myocarditis Fibrinous pericarditis

Fibrinopurulent pericarditis Constrictive pericarditis Pericardial effusion

commonly seen in advanced HIV and other diseases. Clinically significant neutropenia (<500/mm3); highly susceptible to infections (Candida and Aspergillus); most common cause: drug toxicity An increase in the number of white cells in the blood in a variety of inflammatory states caused by microbial and nonmicrobial stimuli that may mimic leukemia.

Enlargement of a lymph node as immune response to foreign antigens; histology usually nonspecific; depends on duration of disease and type of offending agent. Neoplastic white cell disorders Lymphoid neoplasms Most common type of cancer in children; highly aggressive tumors manifesting with signs and symptoms of bone marrow failure, marrow expansion, dissemination of leukemic cells, and CNS manifestations; lymphoblasts with irregular nuclear contours, condensed chromatin, small nucleoli and scant agranular cytoplasm on BMA; blasts compose >25% of marrow cellularity; TdT(+) in 95% of cases; most responsive to chemotherapy (Asparaginase).

Agranulocytosis

Reactive leukocytosis, Leukemoid reaction (high leukocyte alkaline phosphatase, a product of normal WBCs; used to differentiate it from leukemias) Reactive lymphadenitis

Acute lymphoblastic leukemia (ALL) B-cell ALL typically occurs in younger children presenting with BM failure T-cell ALL typically occurs in adolescent males presenting with thymic masses Children 2-10 years old; t(12;21) and hyperdiploidy Male gender; age younger than 2 or older than 10 years; a high leukocyte count at diagnosis; and molecular evidence of persistent disease on day

Tumors Most common tumor of the heart. Metastatic tumor Most common primary tumor of the Myxoma adult heart; major clinical manifestations of this cardiac tumor are due to valvular "ballvalve" obstruction, embolization or a syndrome of constitutional symptoms. The most frequent primary tumor Rhabdomyoma Clinical differences between B- and T-cell of the heart in infants and children; ALL. generally small gray-white myocardial masses composed of a b-all children with BM failure mixed population of cells, the most t-all adolescent male characteristic of which are large, rounded or polygonal cells containing numerous glycogenladen vacuoles separated by strands of cytoplasm, so-called spider cells. Cardiac transplantation Most common indications for DCM and IHD Good prognostic factors in ALL. cardiac transplantation. Primary problem requiring Allograft rejeciton surveillance. Single most important long-term Allograft arteriopathy Poor prognostic factors in ALL. limitation for cardiac (The genetic abnormalities are seen in transplantation. the given age group) CHAPTER 12: DISEASES OF THE WHITE BLOOD CELLS, LYMPH NODES, SPLEEN, AND THYMUS QUESTION ANSWER Non-neoplastic white cell disorders Decreased number of circulating Leukopenia leukocytes; most commonly neutrophils (neutropenia); deficiency of lymphocytes (lymphopenia) is less common, and is TOPNOTCH MEDICAL BOARD PREP PATHO SUPERTABLE Page 18 of 47 For inquiries visit www.topnotchboardprep.com.ph or https://www.facebook.com/topnotchmedicalboardprep/

TOPNOTCH MEDICAL BOARD PREP PATHO SUPERTABLE by KEVIN ELOMINA, MD For inquiries visit www.topnotchboardprep.com.ph or https://www.facebook.com/topnotchmedicalboardprep/

Most common leukemia of adults in Western world; chronic leukemia associated with BCL2, an antiapoptotic molecule; patient presents with increased susceptibility to infections due to hypogammaglobulinemia; CBC showed >5000 lymphocytes/mm3; histologically, foci of mitotically active cells (proliferation centers) are present; also, smudge cells (due to fragility of circulating tumor cells) are also evident; can transform into DLBCL (Richter syndrome). Most common indolent lymphoma of adults; frequent small "cleaved" cells mixed with large cells, growth pattern is nodular, centroblasts present; occurs in older adults, usually involves nodes, marrow, spleen; associated with t(14;18) that results in overexpression of cyclin D1. most common lymphoma of adults; most common form of NHL; tumor cells have large nuclei with open chromatin and prominent nucleoli; most important type of lymphoma in adults, accounting to ~50% of adult NHLs. 17/M presented with a short history of fever, tonsillitis and unilateral enlarged cervical lymph nodes. PE revealed enlargement of right cervical lymph node, 3 cm in diameter, hard, and pharyngeal hyperemia. Histologically, there was intermediate-sized round lymphoid cells with 2-5 prominent nucleoli. High rates of proliferation and apoptosis are characteristic. Nuclear remnants phagocytosed by interspersed macrophages with abundant clear cytoplasm, "starry sky pattern". Also, it is associated with cMYC oncogene [t(8;14)]. The most likely diagnosis is? Clue: it is also the fastest growing human tumor.

Plasma cell neoplasms Disease that presents as multifocal destructive bone lesions seen as punched-out defects on imaging. Renal involvement is also prominent, causing production of proteinaceous casts in the DCT and collecting ducts (that can cause renal insufficiency); can also present with immunodeficiency due to impaired normal plasma cell function. Most common M protein in myeloma cells. Excess light or heavy chains along with complete Igs synthesized by neoplastic plasma cells. Basically, CLL/SLL with plasmacytic differentiation; usually present with

28 of treatment, t(9;22) and MLL rearrangements Chronic lymphocytic leukemia/Small lymphocytic lymphoma (CLL/SLL) SLL: <5000 lymphocytes/m m 3

Follicular lymphoma

Diffuse Large Bcell lymphoma (DLBCL)

Burkitt lymphoma Multiple myeloma

IgG Bence-Jones protein Lymphoplasmac ytic lymphoma

hyperviscosity syndrome (Waldenstrom macroglobulinemia (IgM)): visual changes, neurologic problems, bleeding diathesis and cryoglobulinemia; no bone manifestations; renal manifestations rare. Lymphomas of memory B-cell origin; usually arises on tissues on chronic inflammation; regresses when inciting agent is removed, but once it progresses to DLBCL, regression may not be possible; examples: salivary glands: Sjogren syndrome, thyroid gland: Hashimoto thyroiditis, stomach: H. pylori gastritis These tumors of neoplastic CD4+ T cells home to the skin; usually manifests as a nonspecific erythrodermic rash that progresses over time to a plaque phase and then to a tumor phase; histologically, neoplastic T cells, often with a cerebriform appearance produced by marked infolding of the nuclear membranes, infiltrate the epidermis and upper dermis. Hodgkin lymphomas Diagnostic cells in HL; large, multiple nuclei or single with multiple lobes; each with nucleolus about a size of a small lymphocyte, CD15(+), CD30(+). Classical HL types; CD15 and CD30(+). Most common variant of HL; with lacunar Reed-Sternberg (RS) cells; with deposition of collage bands that divide lymph node into nodules; not associated with EBV; excellent prognosis; usually diagnosed at early stage (Stage I/II) with frequent mediastinal involvement. Mononuclear and diagnostic RS cells with heterogenous cellular infiltrate; associated with EBV in 70% of cases; >50% of cases diagnosed at Stage III/IV . Mononuclear and diagnostic RS cells with lymphocytic infiltrate; not associated with EBV; excellent prognosis. Lymphocytes are scarce with relative abundance of diagnostic RS cells; associated with EBV in 90% of cases; associated with PLHIV; worst prognosis. Lymphohistiocytic "popcorn" RS cells with nodular infiltrate of small lymphocytes admixed with macrophages; excellent prognosis; CD15 and 30 (-), CD20(+). Staging system used for HL.

Myeloid neoplasms BMA shows hypercellular marrow packed with ≥20% myeloblasts (and azurophilic needle-like material called Auer rods (faggot cells)); clinically presents with pancytopenia and bleeding; poor prognosis because it is difficult to treat.

Marginal zone lymphoma

Mycosis fungoides (Cutaneous Tcell lymphoma)

Reed-Sternberg (RS) cells NS, MC, LR, LDHL Nodular sclerosis HL (NSHL)

Mixed cellularity HL (MCHL) Lymphocyterich HL (LRHL) Lymphocytedepleted HL (LRHL) Lymphocytepredominant (LPHL) Ann-Arbor classification Acute myeloid leukemia (AML)

TOPNOTCH MEDICAL BOARD PREP PATHO SUPERTABLE Page 19 of 47 For inquiries visit www.topnotchboardprep.com.ph or https://www.facebook.com/topnotchmedicalboardprep/

TOPNOTCH MEDICAL BOARD PREP PATHO SUPERTABLE by KEVIN ELOMINA, MD For inquiries visit www.topnotchboardprep.com.ph or https://www.facebook.com/topnotchmedicalboardprep/ Stains used to differentiate Myeloblasts from Monoblasts.

Stains used to differentiate Myeloblasts from lymphoblasts. Main differences between AML and ALL.

An AML type that usually presents with DIC; associated with t(15;17) translocations; highly responsive to alltrans retinoic acid. Disorder of defective hematopoietic maturation that results in ineffective hematopoiesis (cytopenias); more common in the elderly; clinically present as bone marrow failure; associated with increased risk of transformation to AML; poor prognosis. Chronic myeloproliferative disorders Common features of chronic myeloproliferative disorders.

Chronic leukemia associated with BCRABL fusion gene (Philadelphia chromosome, t(9;22)); clinically presents with nonspecific symptoms and splenomegaly; BMA shows hypercellular marrow packed with less than 10% myeloblasts with more mature forms; CBC shows leukocytosis >100,000/mm3 with low leukocyte alkaline phosphatase; and scattered macrophages with abundant, wrinkled, green-blue cytoplasm (sea-blue histiocytes); can proceed to a blast crisis if neglected. Increase in all cell lines, but erythroid lines are more increased; associated with JAK2 mutations in most cases; 2% chance of transformation to AML. Increase in megakaryotic lines; associated with JAK2 mutations in 50% of case; transformation to AML is uncommon. Extensive deposition of collagen in marrow by non-neoplastic fibroblasts; associated with JAK2 mutations in 5060% of cases; 5-20% chance of

Myeloblasts: Myeloperoxidas e (MPO)(+) Non-specific esterase (NSE)() Monoblasts: MPO(-), NSE(+) Lymphoblasts: MPO(-) PAS(+) AML occurs in adults, CNS spread is rare, and is more difficult to treat; ALL occurs in children, CNS spread is common and is generally responsive to chemotherapy Acute promyelocytic leukemia Myelodysplastic syndrome

Increased proliferative drive in BM, extramedullary hematopoiesis, Spent phase, and variable transformation to AML Chronic myelogenous leukemia (CML)

Polycythemia vera (PV) Essential thrombocytosis Primary myelofibrosis

transformation to AML. CHAPTER 13: RED BLOOD CELL AND BLEEDING DISORDERS QUESTION ANSWER RED BLOOD CELL DISORDERS Hemolytic anemias Characterized by defective red cell Hereditary membrane proteins spectrin and ankyrin; spherocytosis on PBS, red cells are spherical which lack (Intrinsic; central pallor, and they show increased extravascular) osmotic fragility when placed in hypotonic salt solutions; anemia with characteristically increased MCHC. X-linked disorder in which red cells are Glucose-6unusually susceptible to damage cause by phosphate oxidants. (Drugs that produce oxidants dehydrogenase include antimalarials, sulfonamides, deficiency nitrofurantoin, phenacetin, and vitamin K (G6PD derivatives); Heinz bodies (derived from deficiency) hemoglobin) seen on PBS of (Intrinsic; splenectomized patients; Bite cells seen extravascular on PBS of patients with intact spleen and function. intravascular) This results from substitution of valine Sickle cell for glutamic acid at the 6th position of the anemia β-chain, producing HbS; bizarre, (Intrinsic; elongated, spindled or boat-shaped cells extravascular) on PBS (evident on low oxygen tension states); clinically, prominent cheekbones and changes in skull resembling a "crewcut" skull x-ray, due to bone marrow expansion; also with painful vasoocclusive crisis; spleen may be enlarged, but in chronic cases may be small and fibrosed (autosplenectomy), rendering them susceptible to infections with encapsulated bacteria; treatment is Hydroxyurea that increases HbF. Microcytic, hypochromic anemias Thalassemia characterized by deficiency of either α- or (Intrinsic; β-globin chains (memory device: the extravascular) name is what it lacks). Condition caused by deletion of 1 α- Silent carrier globin gene. Condition caused by deletion of 2 α- α-Thalassemia globin genes; symptomatic at birth because of deficiency in HbF. Disease caused by deletion of 3 alpha α- Hemoglobin H genes. disease Condition caused by deletion of all four α- Hydrops fetalis globin genes. In β-Thalassemia, an individual who β-Thalassemia inherits one abnormal allele (out of 2) minor has this asymptomatic to mildly symptomatic condition; Red cells with a central, dark-red puddle due to collection of hemoglobin (target cells) on PBS. Individuals with β-Thalassemia who β-Thalassemia inherit two abnormal alleles, with severe major (Cooley anemia requiring regular blood anemia) transfusions; usually symptomatic at around six months of age because HbF synthesis ceases at this time; marked hypochromia, poikilocytosis (variation in cell size) and anisocytosis (variation in cell shape) with normoblasts (signifying

TOPNOTCH MEDICAL BOARD PREP PATHO SUPERTABLE Page 20 of 47 For inquiries visit www.topnotchboardprep.com.ph or https://www.facebook.com/topnotchmedicalboardprep/

TOPNOTCH MEDICAL BOARD PREP PATHO SUPERTABLE by KEVIN ELOMINA, MD For inquiries visit www.topnotchboardprep.com.ph or https://www.facebook.com/topnotchmedicalboardprep/ erythropoietic drive). A mutation in the gene pigA causes this disease; there is deficiency of PIG (phosphatidylinositol glycan) that anchors CD55 and CD59 to the red cell surface; without such, red cells become susceptible to complement-mediated lysis. Anemia caused by low-affinity IgM which bind to red cell membranes only at temp <30°C, commonly experienced by distal parts of the body; associated with Mycoplasma pneumoniae, and EBV infections. Hemolysis caused by IgG or IgA antibodies that are active at 37°C, which results in opsonization of red cells by the autoantibodies. Anemia observed in a variety of pathologic states, in which small vessels become particularly obstructed (e.g. DIC, malignant HTN, SLE, etc.); schistocytes, Burr cells, helmet cells, triangle cells are seen in this condition. Anemias of diminished erythropoiesis Most common cause of anemia in both developing and developed countries; microcytic, hypochromic with low ferritin, low serum iron levels low transferrin saturation, and increased TIBC. Most common cause of anemia in hospitalized patients; anemia characterized by high levels of hepcidin, responsible for blocking the transfer of iron to erythroid precursors, hence causing anemia; normocytic, normochromic but can be microcytic, hypochromic; increased ferritin, low serum iron levels, low transferrin saturation, and decreased TIBC. Anemia characterized by hypercellular marrow as a result of increased number of megaloblasts, which are large cells that have delicate, finely reticulated nuclear chromatin and abundant basophilic cytoplasm; can be caused by Folate (without neurologic symptoms) or Cobalamin (with neurologic symptoms) deficiency; PBS findings include: hypersegmented neutrophils, large, eggshaped macro-ovalocytes. Bone marrow is markedly hypocellular, with >90% of the intertrabecular space being occupied by fat; normocytic, normochromic; most commonly caused by chemicals (Benzene) and drugs (Chloramphenicol). Selective suppression of marrow erythroid precursors; can be autoimmune or infectious (Parvovirus) in etiology; presents with normocytic, normochromic anemia with virtuallyt absent eryhtroblasts in bone marrow, and normal granulo- and thrombopoietic elements; paraneoplastic syndrome of

Paroxysmal nocturnal hemoglobinuria (Intrinsic; intravascular) Cold antibody immunehemolytic anemia (Extrinsic; intravascular) Warm antibody immunehemolytic anemia (Extrinsic; intravascular) Microangiopathi c hemolytic anemia (Extrinsic; intravascular)

Iron deficiency anemia

Anemia of chronic disease

Megaloblastic anemia

Aplastic anemia

Pure red cell aplasia (PRCA)

thymoma. Anemia caused by replacement of the bone marrow by in infiltrative processes such as metastatic carcinoma and granulomatous disease; leads to the appearance of early erythroid and granulocytic precursors (leukoerythroblastosis) and teardropshaped red cells (dacrocytes) in the peripheral blood. Denotes an increase in red cells per unit volume of peripheral blood, usually in association with an increase in hemoglobin concentration; maybe relative (hemoconcentration) or absolute; absolute may be primary (polycythemia vera) or secondary (increased erythropoietin, lung disease, high altitude, endurance athletes or erythropoietin-secreting tumors).

BLEEDING DISORDERS Caused by a systemic activation of coagulation pathways, leading to formation of thrombi throughout the microcirculation, and subsequent bleeding due to consumption of coagulation factors; laboratory findings include: thrombocytopenia, increased bleeding time and prolonged PT and PTT. Disorders of platelet number Caused by antiplatelet antibodies directed against glycoproteins IIb-IIIa, IbIX (usually IgG); laboratory findings include: decreased platelet count, prolonged bleeding time, and normal PT and PTT. Form of ITP usually affecting adults and females more than males; associated with SLE, HIV, B-cell neoplasms (CLL); glucocorticoids are indicated; other tretament modalities include, splenectomy, rituximab (anti CD20) in Bcell neoplasms, and TPO-mimetic (Romiplostim). Form of ITP usually affecting children; common antecedent viral illness; glucocorticoids are indicated only when severe; usually self-limited. Thrombocytopenia with thrombosis, in a patient taking anticogulants raises suspicion of the this disorder. Most common hematologic manifestation of HIV infection. Associated with pentad of fever, thrombocytopenia, microangiopathic hemolytic anemia, transient neurologic deficits and renal failure; primary defect is deficiency of ADAMTS13, a vWF metalloproteinase; laboratory findings include: decreased platelet count, prolonged bleeding time, and normal PT and PTT. Associated with childhood onset

Myelophthisic anemia

Polycythemia Disseminated intravascular coagulation (DIC)

Immune thrombocytope nia/Immune thrombocytope nic purpura (ITP) Chronic ITP

Acute ITP

Heparininduced thrombocytope nia (HIT) Thrombocytope nia Thrombotic thrombocytope nic purpura (TTP)

Hemolytic

TOPNOTCH MEDICAL BOARD PREP PATHO SUPERTABLE Page 21 of 47 For inquiries visit www.topnotchboardprep.com.ph or https://www.facebook.com/topnotchmedicalboardprep/

TOPNOTCH MEDICAL BOARD PREP PATHO SUPERTABLE by KEVIN ELOMINA, MD For inquiries visit www.topnotchboardprep.com.ph or https://www.facebook.com/topnotchmedicalboardprep/ microangiopathic hemolytic anemia, thrombocytopenia, and acute renal failure; no neurologic symptoms; often with a history of bloody diarrhea (E. coli O157:H7 infection); with normal ADAMTS13 activity; laboratory findings include: decreased platelet count, prolonged bleeding time, and normal PT and PTT. Disorders of platelet function Decreased gp Ib leads to defective platelet adhesion; laboratory findings include: decreased platelet count, prolonged bleeding time, and normal PT and PTT. Decreased gp IIb-IIIa leads to defective platelet aggregation, laboratory findings include: normal platelet count, prolonged bleeding time, and normal PT and PTT. Coagulation disorders Most common inherited bleeding disorder; most common presenting symptoms of this condition are spontaneous bleeding from mucous membranes, excessive bleeding from wounds, or menorrhagia; bleeding tendency often goes unnoticed until some hemostatic stress, such as surgery, reveals its presence; laboratory findings include normal platelet count, prolonged bleeding time, and normal PT and prolonged PTT. Patient with this disease has a tendency toward easy bruising and massive hemorrhage after trauma or operative procedures, and recurrent bleeding into the joints; laboratory findings include normal platelet count, bleeding time, PT and prolonged PTT. Vitamin K dependent coagulation and fibrinolytic factors, and laboratory picture of Vitamin K deficiency.

Transfusion reactions Transfusion reaction to watch out for in patients with IgA deficiency. IgM-mediated destruction of donor red cells. IgG-mediated destruction of donor red cells in a previously-sensitized individual. ARDS-like complication in patients who underwent transfusion; due to activation of PMNs in lung vasculature.

Uremic Syndrome (HUS)

Bernard-Soulier syndrome

Glanzmann thrombasthenia

Von Willebrand disease (vWD) Hemophilia (Hemophilia A – Factor VIII deficiency; Hemophilia B (Christmas disease) – Factor IX deficiency Factors II, VII, IX, and X; Protein C and S; normal platelet count and bleeding time; prolonged PT and PTT Allergic reactions Acute hemolytic transfusion reaction (AHTR) Delayed hemolytic transfusion reaction (DHTR) Transfusion related acute lung injury (TRALI)

CHAPTER 14: THE LUNG AND PLEURA QUESTION ANSWER

DISEASES OF THE LUNG Loss of lung volume cause by Atelectasis inadequate expansion of airspaces, resulting in shunting of inadequately oxygenated blood from pulmonary arteries into veins; can be resorption (obstruction that prevents air reaching distal air spaces), compression (pleural fluid or mass compressing the lung), and contraction (pleural fibrosis impeding expansion of lungs). Abrupt onset of significant hypoxemia Acute lung injury and bilateral infiltrates WITHOUT (ALI) heart failure. Severe manifestation of ALI; lungs are Acute respiratory dark red, firm, airless and heavy; distress syndrome there is capillary congestion, necrosis (ARDS) of alveolar epithelial cells, interstitial and intra-alveolar edema and hemorrhage and neutrophils in capillaries; hyaline membrane is also characteristic, lining the alveolar ducts. Most common causes of ARDS. Sepsis, diffuse pulmonary infections, Gastric aspiration Class A recommendation for Low Tidal Volume management of ARDS. ventilation: 6cc/Kg PREDICTED Body weight Early neuromuscular blockade: Cisatracurium besylate for 48 hours Obstructive and Restrictive lung diseases Diffuse pulmonary disease Obstructive lung characterized by limitation of airflow, diseases usually resulting from an increase in resistance caused by partial or complete obstruction at any level; spirometry findings: markedly decreased FEV1, decreased FVC, decreased FEV1/FVC; examples: Bronchial asthma, Emphysema, Chronic bronchitis and Bronchiectasis Characterized by abnormal Emphysema (Pink permanent enlargement of the puffers) airspaces distal to the terminal bronchioles accompanied by destruction of their walls without obvious fibrosis; there is thinning of the alveolar walls and loss of elastic tissue; can be centriacinar (respiratory bronchioles only; associated with smoking), panacinar (associated with α-1-antitrypsin deficiency, distal acinar (most commonly seen in adults with

TOPNOTCH MEDICAL BOARD PREP PATHO SUPERTABLE Page 22 of 47 For inquiries visit www.topnotchboardprep.com.ph or https://www.facebook.com/topnotchmedicalboardprep/

TOPNOTCH MEDICAL BOARD PREP PATHO SUPERTABLE by KEVIN ELOMINA, MD For inquiries visit www.topnotchboardprep.com.ph or https://www.facebook.com/topnotchmedicalboardprep/ spontaneous pneumothorax), and irregular (most common). Defined as persistent productive cough for at least 3 consecutive months in at least 2 consecutive years; morphologically, mucosal lining of the larger airways is usually hyperemic and swollen with edema fluid, often covered by a layer of mucinous or mucopurulent secretions; trachea and bronchi have enlarged mucus-secreting glands. Triad of: 1. Intermittent and reversible airway obstruction; 2. Chronic bronchial inflammation with eosinophils; and 3. Bronchial smooth muscle cell hypertrophy and hyperreactivity; morphologically characterized by thick, tenacious mucus plugs with Curschmann spirals (whorls of shed epithelium found in mucus plugs) and Charcot-Leyden crystals (collections of crystalloids made up of eosinophil protein). Criteria for reversibility of airway obstruction diagnostic of bronchial asthma. Permanent dilation of bronchi and bronchioles caused by destruction of the muscle and elastic supporting tissue, resulting from chronic necrotizing infections; morphologically, there is intense acute and chronic inflammatory exudate within the walls, with mixed flora often cultured; there is also peribronchiolar fibrosis in chronic cases. Thickening of airway wall; subbasement membrane fibrosis; increased vascularity in submucosa; increase in size of the submucosal glands and goblet cell metaplasia of the airway epithelium; hypertrophy and/or hyperplasia of the bronchial muscle; are collectively called: Diffuse pulmonary disease characterized by reduced expansion of lung parenchyma accompanied by decreased total lung capacity; spirometry findings: decreased FEV1, markedly decreased FVC, normal or increased FEV1/FVC; examples: Chronic interstitial lung diseases Heterogeneous group of disorders characterized predominantly by inflammation and fibrosis of the pulmonary interstitium; follows restrictive lung disease pattern; complications include cor pulmonale

Chronic bronchitis (Blue bloaters)

Bronchial asthma obstruction inflammation hypertrophy

FEV1 increase of >12% or 200 mL post bronchodilator Bronchiectasis bronchial dilation necrotizing infection (exudate, mixed flora) fibrosis

Airway remodeling

Restrictive lung diseases reduced lung parenchyma, dec FEV1, dec in FVC -> normal to increased. FEV1/VVC

Chronic interstitial lung diseases

and honeycomb lung; examples: Usual interstitial pneumonia (Idiopathic pulmonary fibrosis), Pneumoconiosis, and Sarcoidosis. Pulmonary infections Pneumonia with a patchy distribution of inflammation involving more than one lobe. Pattern of pneumonia observed in pneumococcal pneumonia; has four phases of evolution, from congestion, red hepatization, gray hepatization and resolution. Inflammatory reaction is largely confined within the walls of the alveoli; septa widened and edematous, with mononuclear infiltrates of lymphocytes, histiocytes and plasma cells; alveolar spaces are free of cellular exudate. This morphology points more to: A localized area of suppurative necrosis within the pulmonary parenchyma, resulting in the formation of one or more large cavities; most commonly occurs in the right side, particularly in the posterior segment of the upper lobe and apical segments of the lower lobe; most common isolates are anaerobic bacteria. Tuberculosis Form of tuberculosis that develops in a previously unexposed, unsensitized person. Implantation of the inhaled M. tuberculosis bacilli in the distal airspaces of the lungs, causes the formation of this 1-1.5 cm area of gray-white inflammatory consolidation. Ghon focus with involvement of the hilar lymph nodes is called: Radiographically detectable calcified Ghon complex is called: Pattern of disease that arises in previously sensitized host to M. tuberculosis. Occurs when TB bacilli drain through the lymphatics into lymphatic ducts, which eventually empty into the right side of the heart and into pulmonary circulation; individual lesions are small, visible foci of yellow-white consolidation scattered through the parenchyma. Type of necrosis seen in tuberculosis. Type of inflammation seen in tuberculosis. Type of hypersensitivity reaction in tuberculosis (including TST). Cytokine responsible for formation of granulomas in tuberculosis.

Bronchopneumonia Lobar pneumonia

Atypical pneumonia inflammatory action on walls widened septa mononuclear infiltrates histiocytes and plasma cells free of cellular infiltrates

Lung abscess

Primary tuberculosis Ghon focus

Ghon complex Ranke complex Secondary or reactivation TB Miliary tuberculosis

Caseous Chronic granulomatous Type IV (Cellmediated/Delayed) IFN-γ Macrophages filled with bacilli

Histologic appearance of tuberculosis infection in the absence of an intact cell-mediated immunity. TOPNOTCH MEDICAL BOARD PREP PATHO SUPERTABLE Page 23 of 47 For inquiries visit www.topnotchboardprep.com.ph or https://www.facebook.com/topnotchmedicalboardprep/

TOPNOTCH MEDICAL BOARD PREP PATHO SUPERTABLE by KEVIN ELOMINA, MD For inquiries visit www.topnotchboardprep.com.ph or https://www.facebook.com/topnotchmedicalboardprep/ Most common form of extrapulmonary TB.

Tuberculous lymphadenitis (scrofula) Ileum

Main consequences of gingivitis. Plaque and calculus Fibrous proliferative lesion usually Pyogenic occurring in children, adults and granuloma Most common affected segment in pregnant women (pregnancy tumor); gastrointestinal TB (Note: GITB is histologically, characterized by rare, due to decrease in consumption organizing granulation tissue (highly of infected milk). vascular). Tumors Most common oral fungal infection. Candidiasis One of the two most common Squamous cell Characterized by vesicle formation; Herpetic histologic types of lung cancer carcinoma histologically characterized by gingivostomatitis associated with smoking; centrally ballooned cells with large eosinophilic located in major bronchi; may range intranuclear inclusions (Cowdry Type from poorly differentiated to wellA inclusion bodies). differentiated with keratin pearl A white/red plaque that cannot be Leukoplakia/ formation; associated in general with scraped off; considered precancerous Erythroplakia hypercalcemia, due to production of unless proven otherwise; the red PTH related peptide (PTHrp). plaque is more associated with Immunostains: p63, p40. malignancy than the white plaque; The most common histologic type of Adenocarcinoma most common risk factor is cigarette lung cancer; most commonly seen in smoking. nonsmokers; peripherally located; Neoplasms histologically, may show acinar Most common histologic type of oral Squamous cell (glandular), papillary, mucinous, and cavity cancer. carcinoma solid types. Immunostains: TTF-1 and Most common benign tumor of the Pleomorphic napsin A. salivary gland; histologically adenoma Most common histologic type of lung Small cell (Oat cell) characterized by epithelial elements cancer associated with smoking; carcinoma arranged in ducts, acini, irregular centrally located; with early nodal tubules, strands, or even sheets, in a involvement; histologically, small, mesenchyme-like background of loose round to fusiform cells with scant myxoid tissue containing islands of cytoplasm and finely granular chondroid and rarely, foci of bone; chromatin with numerous mitotic locally invasive. figures are present; most associated The most common primary malignant Mucoepidermoid with paraneoplastic syndromes, such tumor of salivary glands composed of carcinoma as Cushing syndrome (ectopic ACTH), mucous and squamous cells. Lambert-Eaton myasthenic syndrome, Most common epithelial odontogenic Ameloblastoma and SIADH, among others; only type tumor that arises from odontogenic not amenable to surgery; epithelium that does not demonstrate Immunostains: chromogranin, extomesenchymal differentiation; synaptophysin, CD57 and BCL2. locally invasive. Lung carcinoma that typically has Large cell DISEASES OF THE UPPER AIRWAYS large, prominent nucleoli, and a carcinoma Rare neoplasm with strong links to Nasopharyngeal moderate amount of cytoplasm, with EBV and high frequency among the carcinoma minimal glandular or squamous Chinese population. Characterized by differentiation. Diagnosed when large epithelial cells having indistinct (Note: some types immunostains for known histologic borders (syncytial growth) and of HL, and Burkitt types of lung cancer are negative. prominent eosinophilic nucleoli. are also associated DISEASES OF THE PLEURA with EBV) A rare cancer seen in patients who Malignant A benign neoplasm usually on the true Laryngeal work in shipyards, miners and mesothelioma vocal cords that forms a soft, papilloma insulators; presents as lung raspberry-like excresence rarely more ensheathed by a yellow-white, firm or (Note: Malignant then 1 cm in diameter; consists of gelatinous layer of tumor that mesothelioma is multiple, slender, finger-like obliterates the pleural space; lesion is commonly projections. preceeded by extensive pleural associated with Most common symptom of laryngeal Persistent fibrosis and plaque formation; may be asbestos, but the carcinoma. hoarseness epithelial, sarcomatoid or biphasic in most common Common etiologic agents implicated Smoking, alcohol, morphology. cancer asbestos in laryngeal carcinoma asbestos, radiation, Immunostains: Epithelioid: Keratin, causes is still lung HPV calretinin, WT-1, CK5,6, D2-40; cancer) Most common histologic type of Squamous cell Sarcomatoid: usually Keratin(+) only. laryngeal carcinoma carcinoma (95%) Most common location of laryngeal Glottic (60-75%) CHAPTER 15: HEAD AND NECK carcinoma QUESTION ANSWER EARS ORAL CAVITY Most common cause of otitis media. Viral Non-neoplastic conditions Most common primary bacterial S. pneumoniae, nonMain cause of tooth loss before age Dental caries causes of acute otitis media. typeable H. 35. influenzae, M. TOPNOTCH MEDICAL BOARD PREP PATHO SUPERTABLE Page 24 of 47 For inquiries visit www.topnotchboardprep.com.ph or https://www.facebook.com/topnotchmedicalboardprep/

TOPNOTCH MEDICAL BOARD PREP PATHO SUPERTABLE by KEVIN ELOMINA, MD For inquiries visit www.topnotchboardprep.com.ph or https://www.facebook.com/topnotchmedicalboardprep/ Most common causes of chronic otitis media. Complications of otitis media. NECK True cyst arising from remnant of second branchial arch (most common); clinically presents as upper lateral neck mass along SCM.

catarrhalis P. aeruginosa, S. aureus, fungus, mixed Brain abscess and cholesteatoma Branchial cleft cyst Thyroglossal duct cyst

metaplasia within the esophageal squamous mucosa; increased risk of esophageal adenocarcinoma. Neoplasms Esophageal carcinoma associated with chronic GERD; usually occurs in the distal third of the esophagus; histologically characterized by gland formation and mucin production; premalignant lesion is Barrett esophagus. More common of the histologicpatterns of esophageal cancer; associated with smoking, alcohol, caustic injury, and frequent consumption of very hot beverages; usually occurs in the middle third of the esophagus; premalignant lesion is squamous dysplasia.

Adenocarcinoma

Squamous cell carcinoma

Most common congenital anomaly of thyroid gland; arising from remnants of thyroid gland descent; clinically present as a midline mass along the track of descent of the thyroid gland; treatment of choice is Sistrunk procedure. Neuroendocrine neoplasm associated Paranganglioma with ANS; basically, an extra-adrenal pheochromocytoma (histologically); STOMACH can be located paravertebral Gastritides (sympathetic), or in aorticopulmonary chain (parasympathetic). Most common form of chronic H. pylori gastritis gastritis; most common cause of SALIVARY GLANDS multifocal atrophic gastritis; Most common type of inflammatory Mucocele characterized by the following: salivary gland lesion and the most predominantly antral, neutrophilic common salivary gland lesion. infiltrate with subepithelial plasma Most common form of viral Mumps cells, serum gastrin normal to sialadenitis; most common gland decreased; associated with involved: parotid. hyperplastic/inflammatory polyps; Most common gland involved in Submandibular (+) for H. pylori antibodies; sialolithiasis. sequelae includes gastric Most common site of tumors Parotid gland lymphoma. regardless of behavior. Most common form of gastritis in Autoimmune gastritis Most common tumor of the salivary Pleomorphic patients without H. pylori infection; gland; also the most common benign adenoma most common cause of diffuse tumor. atrophic gastritis; characterized by Most common primary malignant Mucoepidermoid the following: predominantly at the tumor of the salivary gland; and the carcinoma body, lymphocytic infiltrate with second most common tumor. macrophages, serum gastrin Second most common benign tumor Warthin tumor increased; associated with of the salivary gland, with high (Papillary neuroendocrine hyperplasia; (+) association with smoking. cystadenoma for antibodies against parietal cells lymphomatosum) (proton pump and intrinsic factor); Highly aggressive malignancy of the Adenoid cystic clinically, patients present with salivary gland, with high propensity carcinoma achlorhydia (due to destruction of for perinueral invasion, and high parietal cells), and megaloblastic occurrence of distant metastasis. anemia (due to Vitamin B12 deficiency). CHAPTER 16: THE GASTROINTESTINAL TRACT Peptic ulcer disease ESOPHAGUS Most common form of PUD. Antral or duodenal (H. Non-neoplastic conditions pylori) Typical findings in significant Eosinophilic infiltrate Histologic findings in PUD. NIGS: N: necrosis, I: GERD. in mucosa with acute inflammation, neutrophils (if severe) G: granulation tissue, and Basal zone S: fibrous scar (from hyperplasia superficial to deep) Portocaval anastomoses involved Esophageal branch of Neoplasms in esophageal varices. left gastric vein and Most common gastric polyp. Inflammatory/ azygos vein Hyperplastic (75%) Complication of long-standing Barrett esophagus The most common malignancy of Gastric GERD; characterized by intestinal TOPNOTCH MEDICAL BOARD PREP PATHO SUPERTABLE Page 25 of 47 For inquiries visit www.topnotchboardprep.com.ph or https://www.facebook.com/topnotchmedicalboardprep/

TOPNOTCH MEDICAL BOARD PREP PATHO SUPERTABLE by KEVIN ELOMINA, MD For inquiries visit www.topnotchboardprep.com.ph or https://www.facebook.com/topnotchmedicalboardprep/ the stomach (90%); has two types: intestinal (bulky tumors with gland-like structures histologically); and diffuse (intense desmoplasia leading to leather bottle stomach (linitis plastic) with signet ring cells containing mucin histologically. (Lauren classification of intestinal and diffuse is used); most common site: Antrum, on the lesser curvature. Malignancy associated with H. pylori infection; most common site of occurrence is the stomach; eradication of H. pylori leads to tumor regression, except when it transforms to DLBCL. Tumors of neuroendocrine origin; common site is small intestine; may elaborate hormones that can lead to ZES (gastrin) or carcinoid syndrome (serotonin); most important prognostic factor is location i.e. midgut tumors are often multiple and aggressive; immunostains: Synaptophysin and Chromogranin A. Most common mesenchymal tumor of the abdomen; more than half of cases occur in the stomach; cytogenetic origin is interstitial cells of Cajal (ICC); histologically composed of elongated spindle cells or plump epithelioid cells; Immunostains: KIT.

adenocarcinoma

Lymphoma/MALToma

Carcinoid

Gastrointestinal stromal tumor (GIST) SMALL AND LARGE INTESTINES Intestinal obstruction Most common cause of intestinal Hernias obstruction. Most common cause of intestinal Intussusception obstruction in children <2 years old. Ischemic bowel disease Most common regions susceptible Splenic flexure and to ischemia. rectosigmoid Inflammatory bowel disease Inflammatory bowel disease Crohn disease characterized by the following: skip lesions, transmural bowel involvement, and rectal sparing; associated with development of colonic adenocarcinoma if with colonic involvement. Inflammatory bowel disease Ulcerative colitis characterized by the following: continuous lesions usually limited up to submucosa, with rectal involvement but disease is limited to the colon only; associated with toxic megacolon and development of colonic adenocarcinoma. Note: Make sure to study Table 14-5 in Robbins Basic Pathology, 9th ed. p. 587 or Table 17-8 in Robbins and Cotran Pathologic Basis of Disease 8th ed. p. 808 or Table 17-9 in Robbins and Cotran Pathologic Basis of Disease 9th ed. p. 797. Diverticular disease Most common site of diverticular Sigmoid (in areas

disease. Type of diverticulum in sigmoid .diverticulosis. Neoplasms Polyps common in the left colon; characterized by serrated architecture limited to the upper third of the crypt; no cytologic atypia and lateral growth; do not have malignant potential. Polyps common in the right colon; characterized by serrated architecture throughout the whole length of the gland; no cytologic atypia but exhibits lateral growth (elephant-feet glands); has malignant potential like conventional adenomas. Autosomal dominant syndrome of hamartomatous polyps and mucocutaneous hyperpigmentation, associated with increased risk of visceral malignancies; histologically characterized by arborizing networks composed of smooth muscle intermixed with lamina propria. Morphologic variants of colonic adenomas. The single most important characteristic of adenomas that correlates with malignancy. Familial syndrome characterized by early onset development of numerous colonic polyps; main pathology is mutations on the APC gene (Ch5); criterion is the presence of at least 100 polyps.

Familial syndrome characterized by early development of colon cancer and other visceral malignancies; main pathology is mutations in genes of DNA repair (MSH2 or MLH1) that leads to microsatellite instability and subsequent development of colonic adenocarcinoma. Most common malignancy of the gastrointestinal tract; two pathways are recognized; the adenoma carcinoma sequence involving APC among others; and the microsatellite instability pathway (sessile serrated adenoma-carcinoma sequence) involving MSH2 or MLH1; clinically characterized by different symptomatologies depending on location: (Right colon: usually anemia from bleeding from the

penetrated by blood vessels) False diverticulum

Hyperplastic polyp

Sessile serrated adenoma

Peutz-Jeghers syndrome

Tubular, Tubulovillous, Villous Size (>4cm) (others: architecture (villous), and severity of dysplasia Familial adenomatous polyposis (FAP) Hereditary nonpolyposis colon cancer (HNPCC)/Lynch syndrome

Colonic adenocarcinoma

TOPNOTCH MEDICAL BOARD PREP PATHO SUPERTABLE Page 26 of 47 For inquiries visit www.topnotchboardprep.com.ph or https://www.facebook.com/topnotchmedicalboardprep/

TOPNOTCH MEDICAL BOARD PREP PATHO SUPERTABLE by KEVIN ELOMINA, MD For inquiries visit www.topnotchboardprep.com.ph or https://www.facebook.com/topnotchmedicalboardprep/ bulky nonobstructive masses; Left liver disease and portal hypertension hypertension colon: usually presents with change due to excessive pulmonary vascular in bowel habits due to napkin-ring remodeling; Clinically presents with configuration of the lesion). exertional dyspnea and clubbing of the fingers. Two important prognostic factors Invasion and lymph in colonic adenocarcinoma. node status Clinical entity that occurs when Jaundice retention of bilirubin leads to serum Most common site of metastases of Liver levels of at least 2.0-2.5 mg/dL. colonic adenocarcinoma. Most common cause of conjugated Hepatitis and intra- APPENDIX hyperbilirubinemia. and extrahepatic The most common cause of acute Lymphoid hyperplasia obstruction of bile appendicitis in children and adults, and Fecalith, flow respectively. respectively Most common cause of unconjugated Hemolysis The most common tumor of the Carcinoid hyperbilirubinemia. appendix. Hepatitides Note: Please study Table 15-5 of Robbins Basic Pathology, 9th CHAPTER 17: LIVER AND GALLBLADDER ed p. 613, or Table 18-3 of Robbins and Cotran Pathologic QUESTION ANSWER Basis of Disease, 9th ed p. 836, for the detailed morphologies of LIVER acute and chronic hepatitides. Clinical syndromes The main differences in morphology Acute hepatitides Condition wherein there is 80-90% Acute liver failure of acute and chronic hepatitides. have less liver parenchymal loss in patient inflammation and without pre-existing liver disease; more hepatocyte clinically presents with death than chronic encephalopathy and coagulopathy; hepatitis occurs <26 weeks post initial insult. Which virus between HBV and HCV is HCV (most cases of A clinical entity caused by severe loss Hepatic more associated with hepatocellular acute HBV infection of hepatoceullular function and encephalopathy carcinoma? present with shunting of blood from portal to subclinical disease systemic circulation, causing buildup and all of them of toxins that cause neuronal recover) dysfunction (in acute cases); Morphologic appearance of chronic Ground glass manifestations may range from Hepatitis B infection. hepatocytes subtle behavioral changes to deep Morphologic appearance of chronic Lymphoid follicles, coma and death. Hepatitis C infection. bile duct injury and Syndrome heralded by a drop in Hepatorenal steatosis urine output and rising BUN and syndrome Hepatitis characterized by early Autoimmune creatinine; appears in severe liver development of scarring after the hepatitis failure; characterized by renal failure initial wave of hepatocyte injury in the setting of liver disease without (versus viral hepatitides where primary renal abnormalities scarring usually happens years after Common causes of chronic liver Chronic HBV and the initial insult); associated with failure. HCV, Alcoholic liver confluent necrosis, marked disease, and NAFLD inflammation, plasma cell infiltrate, Diffuse transformation of the liver Cirrhosis and burned-out cirrhosis; has two into regenerating parenchymal types: Type 1 (in adults; associated nodules, surrounded by dense bands with ANA and anti-SMA); and Type 2 of scar, with variable degrees of (in children; associated ithe Antivascular shunting. LKM1). Common causes of death in chronic Complications of Most common drug causing ALF. Acetaminophen liver failure. acute liver failure Most common substance causing CLF. Alcohol and hepatocellular Alcoholic and Non-alcoholic fatty liver disease carcinoma Amount of alcohol associated with 80 g/d development of ALD. Liver enzyme pattern unique to ALD. AST > ALT Three morphological forms of Hepatocellular Increased resistance to portal blood Portal hypertension alcoholic liver disease. steatosis; Alcoholic flow that may be caused by Steatohepatitis; and prehepatic, intrahepatic and Steatofibrosis posthepatic causes; can cause dilation Hepatocyte ballooning, eosinophilic Alcoholic of the portocaval anastomoses and intarcytoplasmic inclusion bodies Steatohepatitis produce varices. composed of intermediate filaments Severe arterial hypoxemia with Hepatopulmonary (Mallory-Denk bodies) and dyspnea and cyanosis in patients syndrome neutrophilic infiltration are with liver disease due to abnormal characteristic of: intrapulmonary vascular dilatation and increased pulmonary blood flow. Most common metabolic liver NAFLD Pulmonary hypertension arising in Portopulmonary disease; usually occurs in patients TOPNOTCH MEDICAL BOARD PREP PATHO SUPERTABLE Page 27 of 47 For inquiries visit www.topnotchboardprep.com.ph or https://www.facebook.com/topnotchmedicalboardprep/

TOPNOTCH MEDICAL BOARD PREP PATHO SUPERTABLE by KEVIN ELOMINA, MD For inquiries visit www.topnotchboardprep.com.ph or https://www.facebook.com/topnotchmedicalboardprep/ consuming <20 g/week of ethanol; associated with Type 2 DM, insulin resistance, dyslipidemia, and hypertension. Most common outcome of NAFLD

Difference of non-alcoholic fatty liver disease from ALD. A rare disease characterized by microvesicular fatty change in the liver and encephalopathy; microscopy of hepatocellular mitochondria reveals pleomorphic enlargement and electron lucency of the matrices, with disruption of cristae and loss of dense bodies; clinically suspected in a pediatric patient who took Aspirin as antipyretic for a viral infection. Inherited metabolic diseases Triad of micronodular cirrhosis, diabetes mellitus and abnormal skin pigmentation; main pathology is extensive accumulation of body iron in the liver, pancreas and heart; most common form is autosomal recessive, involving mutations in HFE gene in Chromosome 6; associated with 200fold increased risk of HCC. Extensive accumulation of toxic levels of Copper in the liver, brain and eye; (Kayser-Fleischer rings); autosomal recessive; main pathology is loss of function mutations in ATP7B gene in Chromosome 13 Autosomal recessive disorder characterized by panacinar emphysema and liver disease secondary to accumulation of misfolded proteins; histologically marked by PAS-positive, diastaseresistant, cytoplasmic inclusions, corresponding to mutant AAT; associated with increased risk of HCC. Cholestatic liver diseases Most common cause of large bile duct obstruction in adults, and children, respectively. Secondary bacterial infection of biliary tree; clinically typified by Charcot triad: 1. fever, 2. jaundice, and 3. RUQ pain. Intrehepatic gallstone formation (calcium bilirubinate) due to repeated bouts of ascending cholangitis and parenchymal destruction; associated with increased risk of cholangiocarcinoma. Prolonged conjugated hyperbilirubinemia in the neonate; can either be caused by extrahepatic biliary atresia (can be corrected with surgery), or neonatal hepatitis

(histologically characterized by multinucleated giant cells, and cannot be corrected, or even worsened with surgical intervention). Isolated fatty liver (>80%); most cases do not progress to NASH Less prominent features of steatohepatitis compared to ALD Reye syndrome

Hereditary Hemochromatosis

Wilson disease (Hepatolenticular degeneration)

α-1-antitrypsin deficiency

Gallstones, biliary atresia Ascending cholangitis Primary hepatolithiasis

Neonatal cholestasis

Extrahepatic biliary atresia

Complete or partial obstruction of the lumen of the extrahepatic biliary tree within the first 3 months of life; single most common cause of death from liver disease in early childhood; most common is perinatal form; treatment of choice is Kasai procedure. Autoimmune cholangiopathies A chronic, progressive cholestatic Primary biliary liver disease characterized by cirrhosis (PBC) nonsuppurative destruction of small to medium-sized intrahepatic bile ducts; associated with antimitochondrial antibodies is 90% of cases; histologically, characterized by destruction of interlobular bile ducts with lymphoplasmacytic infiltration with or without granulomas (florid duct lesions). Chronic progressive fibrosis and Primary sclerosing destruction of extrahepatic and cholangitis (PSC) intrahepatic bile ducts of all sizes; patchy involvement gives rise to a beading appearance of ducts; histologically characterized by onionskin lesion with an atrophic tubule on the center (small ducts) or acute on chronic inflammation (large ducts); associated with an increased risk of cholangiocarcinoma. Structural abnormalities of the biliary tree Triad of choledochal cysts. Pain, jaundice, abdominal mass Choledochal cysts and fibropolycystic Cholangiocarcinoma diseases predisposes to this particular kind of cancer. Tumors Most common benign tumor of the Cavernous liver. hemangioma Most common liver tumor of early Hepatoblastoma childhood. A well-demarcated but poorly Focal nodular encapsulated lesion, consisting of hyperplasia hyperplastic hepatocyte nodules with a central fibrous scar; carries no risk for malignancy. A benign tumor characterized by Hepatic adenoma hepatocytes without portal tracts with prominent neovascularization; harbors a risk for malignant transformation. Type of hepatocellular adenoma with β-catenin adenomas the highest risk of malignant transformation. Most common primary malignant Hepatocellular tumor of the liver; commonly arises carcinoma in the setting of chronic liver disease; may appear as unifocal, multifocal or diffusely infiltrative; with strong

TOPNOTCH MEDICAL BOARD PREP PATHO SUPERTABLE Page 28 of 47 For inquiries visit www.topnotchboardprep.com.ph or https://www.facebook.com/topnotchmedicalboardprep/

TOPNOTCH MEDICAL BOARD PREP PATHO SUPERTABLE by KEVIN ELOMINA, MD For inquiries visit www.topnotchboardprep.com.ph or https://www.facebook.com/topnotchmedicalboardprep/ propensity for vascular invasion; morphological changes: 1. histologically, well-differentiated microvascular leak and edema; 2. lesions may elaborate bile appearing enzymatic fat necrosis; 3. acute as globules; tumor marker is AFP. inflammation; 4. destruction of pancreatic parenchyma; and 5. Most common site of hematogenous Lung destruction of blood vessels and metastases of HCC. interstitial hemorrhage. Most common tumor involving the Metastases Most common causes of acute Alcoholism and liver. pancreatitis. biliary tract disease GALLBLADDER (80%) Gallstones Prolonged inflammation of the Chronic Most common type of gallstones; pale Cholesterol stones pancreas associated with irreversible pancreatitis yellow in color; radiolucent. (80%) destruction of exocrine parenchyma, Pigment stones derived from sterile Black pigment fibrosis, and, in the late stages, the gallbladder bile; 50-70% are stones destruction of endocrine parenchyma. radiopaque. Most common cause of chronic Long-term alcohol Pigment stones derived from infected Brown pigment pancreatitis. abuse gallbladder bile; radiolucent. stones More common form of cystic disease Pseudocysts in the pancreas. Complication of cholelithiasis, Gallstone ileus Neoplasms wherein a large stone erodes directly (Bouveret Most common site of pancreatic Head (presents as into adjacent small bowel. syndrome) adenocarcinoma. obstructive Most common form of acute Acute calculous jaundice) cholecystitis. cholecystitis (90%) Most common mutated oncogene in KRAS The gallbladder may be contracted, of Chronic acalculous pancreatic adenocarcinoma. normal size, or enlarged; presence of cholecystitis Most common mutated tumor p16 stones in the absence of inflammation suppressor gene in pancreatic is diagnostic. adenocarcinoma . Most common form of chronic Chronic calculous Strongest environmental risk factor Smoking cholecystitis; characterized by cholecystitis for developing pancreatic mononuclears in GB wall, with (>90%) adenocarcinoma. Rochitansky-Aschoff sinus. Two distinct features of pancreatic Highly invasive and Tumors ductal adenocarcinoma. intense Most common malignant tumor of the Gallbladder desmoplasia biliary tract; maybe exophytic (with carcinoma Tumor marker for pancreatic CA19-9 mass) or infiltrating (with thickening adenocarcinoma. of the wall; more common); most The usual sites of metastases of Liver and lungs frequent histology is pancreatic adenocarcinoma. adenocarcinoma. Paraneoplastic syndrome associated Trosseau syndrome Most important risk factor for the Gallstones with pancreatic adenocarcinoma, development of gallbladder characterized by migratory superficial carcinoma. thrombophlebitis, secondary to Second most common primary Cholangiocarcinoma expression of pro-coagulant factors malignant tumor of the liver; from the tumor or its necrotic malignant tumor of cholangiocytes; products. more common in extrahepatic bile ducts; may develop in the hilum CHAPTER 19: THE KIDNEY (Klatskin tumor); risk factors include: NAFLD, PSC, fibrocystic diseases of QUESTION ANSWER the biliary tree (choledocal cysts), Glomerular diseases and infestation of Clonorchis sinensis Nephritic syndrome: 1. Hematuria (with dysmorphic RBCs and Opistorchis viverrini; most and red cell casts indicating glomerular pathology); 2. frequent histology is Oliguria and azotemia; and 3. Hypertension adenocarcinoma. Most common cause of nephritic Acute syndrome in children Postinfectious CHAPTER 18: THE PANCREAS LM: diffuse hypercellularity (almost, if (Poststreptococcal) QUESTION ANSWER not all glomeruli) Glomerulonephritis Congenital anomalies EM: subepithelial humps on GBM (PSAGN) IF: granular deposits of IgG and What is the most common clinically Pancreatic divisum complement within the capillary walls significant congenital anomaly of the mesangium pancreas? Rapidly progressing glomerulonephritis: syndrome of Pancreatitides progressive loss of renal function, characterized by Reversible pancreatic parenchymal Acute pancreatitis nephritic syndrome often with severe oliguria; histologic injury associated with inflammation; hallmark is presence of “crescents”, thus crescentic GN pathology is inappropriate release Type of RPGN associated with antiRPGN Type I and activation of pancreatic enzymes, GBM antibodies; associated with which destroy pancreatic tissue and Goodpasture syndrome (glomerular elicit an acute inflammatory reaction; and pulmonary involvement); characterized by the following TOPNOTCH MEDICAL BOARD PREP PATHO SUPERTABLE Page 29 of 47 For inquiries visit www.topnotchboardprep.com.ph or https://www.facebook.com/topnotchmedicalboardprep/

TOPNOTCH MEDICAL BOARD PREP PATHO SUPERTABLE by KEVIN ELOMINA, MD For inquiries visit www.topnotchboardprep.com.ph or https://www.facebook.com/topnotchmedicalboardprep/ uninvolved segments shows no proliferation EM: ruptures in the GBM IF: linear IgG and C3 deposits along the GBM Type of RPGN associated with RPGN Type II immune complex deposition; usually a secondary event of immune complexmediated nephritides; uninvolved segment shows immune complex deposition EM: “lumpy bumpy” appearance of GBM (due to deposits) IF: granular deposition of Ig and complement in GBM Type of RPGN associated with ANCA; RPGN Type III sometimes a component of ANCA vasculitides (Microscopic polyangiitis and Wegener granulomatosis); uninvolved segments shows no proliferation EM: no detectable deposits IF: negative for Ig and complement An important cause of ESRD; grossly, Chronic kidneys are symmetrically contracted, glomerulonephritis surfaces are red-brown and diffusely granular; histologically, glomeruli are obliterated with marked interstitial fibrosis. Nephrotic syndrome: 1. Massive proteinuria (≥3.5g/dL; 2. Hypoalbuminemia (<3g/dL); 3. Generalized edema; 4. Hyperlipidemia and lipiduria Most common cause of nephrotic Minimal change syndrome in children; responsive to disease/Lipoid corticosteroids. nephrosis LM: none EM: uniform and diffuse effacement of foot processes of the podocytes Most common cause of nephrotic Focal segmental syndrome in adults; involves only glomerulosclerosis some glomeruli (focal), and only a (FSGS) part of glomerulus (segmental) is affected; associated with HIV and heroin abuse. LM: increased mesangial matrix, obliterated capillary lumina, and deposition effacement of foot processes of hyaline masses (hyalinosis) and lipid droplets. EM: IF: nonspecific trapping of immunoglobulins, usually IgM, and complement in the areas of hyalinosis. Associated with infections (HBV, Membranous Syphilis, Schistosomiasis, Malaria), nephropathy Malignant solid tumors (lung and colon), SLE, Gold and Mercury, and drugs (Penicillamine, Captopril, NSAIDs); LM: diffuse thickening of the capillary wall EM: subepithelial deposits along the GBM (spike and dome appearance); effacement of foot processes IF: granular deposits of immunoglobulins and complement along the GBM Associated with HBV, HCV, SLE, and Membranoprolifera

infected AV shunts LM: Thickened split GBM (Tram track appearance) EM: subendothelial electron-dense deposits IF: Irregular granular C3 deposits, with IgG and early complement components (C1q and C4) Formerly called MPGN Type II; fundamental abnormality is excessive complement activation LM: Thickened split GBM (Tram track appearance) EM: lamina densa and glomerular capillary wall transformed into irregular, ribbon-like, extremely electron-dense structure IF: Irregular chunky and segmental linear foci of C3 deposits in GBM and mesangium, without IgG and early complement components (C1q and C4) Isolated glomerular abnormalities One of the most common causes of recurrent microscopic or gross hematuria; most common glomerular disease revealed by renal biopsy worldwide. LM: mesangial widening and segmental inflammation EM: mesangial electron-dense deposits IF: mesangial deposition of IgA, often with C3 and properdin and smaller amounts of IgG or IgM

tive glomerulonephritis (MPGN) Type I

Dense deposit disease

IgA nephropathy (Berger disease) Note: Berger disease and Henoch-Schonlein purpura have the sam renal manifestations and morphology, but HSP involves systemic deposition of IgA, has extrarenal symptoms Nephritis associated with hearing and Alport syndrome visual defects; defect in Type IV collagen synthesis; X-linked dominant pattern of inheritance LM: glomerulosclerosis, vascular sclerosis, tubular atrophy, and interstitial fibrosis EM: thin GBM (early); irregular foci of GBM thickening or attenuation (late); “basket-weave” appearance Diseases of the Tubules and Interstitium: Tubulointerstitial nephritides Clinicopathologic entity characterized Acute tubular clinically by acute renal failure and injury/necrosis often, but not invariably, morphologic evidence of tubular injury, in the form of necrosis of tubular epithelial cells; has two forms: ischemic and toxic, which vary in terms of affected segments of the nephron and distribution of necrosis. Most common cause of acute kidney Acute injury; a common suppurative pyelonephritis inflammation of the kidney and renal pelvis caused by bacterial infection, either by hematogenous spread or through ascending infection (more common and more important mode); principal causes are Gram-negative enteric rods; grossly, there is discrete,

TOPNOTCH MEDICAL BOARD PREP PATHO SUPERTABLE Page 30 of 47 For inquiries visit www.topnotchboardprep.com.ph or https://www.facebook.com/topnotchmedicalboardprep/

TOPNOTCH MEDICAL BOARD PREP PATHO SUPERTABLE by KEVIN ELOMINA, MD For inquiries visit www.topnotchboardprep.com.ph or https://www.facebook.com/topnotchmedicalboardprep/ yellowish, raised abscesses grossly apparent on the renal surface; histologically, there is liquefactive necrosis with abscess formation within the renal parenchyma. Hallmark of this disease is scarring involving the pelvis or calyces, or both, leading to papillary blunting and marked calyceal deformities; an important cause of chronic kidney disease; forms include: reflux nephropathy (more common), and chronic obstructive pyelonephritis. Second most common cause of acute kidney injury; T-cell mediated immune reaction of the kidneys to an offending agent, characterized by interstitial inflammation, with abundant eosinophils and edema. Vascular diseases Form of nephrosclerosis associated with essential hypertension; histologically, hyaline arteriolosclerosis; renal insufficiency is uncommon except in patients of African descent, severe HPN, and other diseases e.g. DM. Form of nephrosclerosis associated with malignant hypertension; histologically, hyperplastic nephrosclerosis and fibrinoid necrosis. Most common causes of renal artery stenosis. Nephron segments most susceptible to ischemia, and therefore, most affected in ischemic ATN. Cystic diseases of the kidney Autosomal dominant; kidneys are enlarged, composed solely of cysts without intervening parenchyma; cysts are filled with clear or turbid fluid; cysts may arise at any level of the nephron, with variable, often atrophic lining; pathology is defective gene PKD1, which codes for polycystin-1; a protein that is involved in cell-cell or cell-matrix adhesion; most common cause of death: CAD or HHD (40%).

Autosomal recessive; numerous small cysts in the cortex and medulla, giving the kidney a "sponge-like" appearance; cysts have uniform cuboidal epithelium; associated with multiple cysts in the liver; pathology is defective PKHD1, which codes for fibrocystin; a protein found in cilia in tubular epithelial cells. Nephrolithiasis

Most common composition of kidney stones.

Chronic pyelonephritis

Drug-induced interstitial nephritis

Nephrosclerosis

Malignant nephrosclerosis

Atherosclerosis and fibromuscular dysplasia Short; straight segments of PT and AL LOH Adult Polycystic Kidney Disease (ADPKD) Childhood Polycystic Kidney Disease (ARPKD)

Most important cause of kidney stone formation. Kidney stones occurring in patients with alkaline urine due to UTI, particularly Proteus vulgaris and staphylococci; most common composition of staghorn calculi. Kidney stones seen in patients with gout and leukemias; urine pH is decreased; radiolucent. Kidney stones associated with a defect in the renal transport of certain amino acids; forms in acidic urine. Tumors Tumors derived from renal tubular epithelium, located primarily at the cortex; with three common forms, clear cell, papillary renal cell and chromophobe carcinomas; most common primary malignant tumor of the kidney. Most important risk factor for renal cell carcinoma. Most common form (70-80%) of renal cell carcinoma and associated with homozygous loss of the VHL tumor suppressor gene; thought to arise from proximal tubule cells; usually solitary and large, spherical masses reaching up to 15 cm in diameter; cut surface shows yellow orange to graywhite masses, with prominent areas of cystic softening and hemorrhage; cells appear vacuolated or may be solid; often invades the renal vein. Accounts for 10-15% of cases of RCC; throught to arise from distal tubule cells; characterized by varying degrees of papilla formation with fibrovascular cores; cells have clear to pink cytoplasm; associated with increased activity of MET oncogene; tends to be bilateral. Accounts for 5% of cases of RCC; thought to arise from intercalated cells of collecting ducts; grossly tanbrown; microscopically, cells have clear, flocculent cytoplasm with very prominent, distinct cell membranes; nuclei surrounded by halos of cleared cytoplasm. Most common site of metastases of RCC.

Calcium oxalate and/or Calcium phosphate (80%) Supersaturation Struvite (Magnesium ammonium phosphate); Uric acid stones Cystine stones

Renal cell carcinoma (RCC)

Smoking Clear cell RCC

Papillary RCC

Chromophobe RCC

Lungs (>50%), followed by bones (33%)



TOPNOTCH MEDICAL BOARD PREP PATHO SUPERTABLE Page 31 of 47 For inquiries visit www.topnotchboardprep.com.ph or https://www.facebook.com/topnotchmedicalboardprep/

TOPNOTCH MEDICAL BOARD PREP PATHO SUPERTABLE by KEVIN ELOMINA, MD For inquiries visit www.topnotchboardprep.com.ph or https://www.facebook.com/topnotchmedicalboardprep/ CHAPTER 20: THE LOWER URINARY TRACT AND MALE GENITAL SYSTEM QUESTION ANSWER Penis An abnormal opening of the urethra Hypospadia (more along the ventral or dorsal aspect of common); the penis, respectively. epispadia Inflammation of the glans and of Balanitis; overlying prepuce, respectively; cause balanoposthitis by Gardnerella, Candida, anaerobic and pyogenic bacteria. Appears grossly as a solitary, plaqueBowen like lesion on the shaft of the penis; disease/Squamous histologic examination reveals cell carcinoma in morphologically malignant cells situ of the penis throughout the epidermis with no invasion of the underlying stroma; has potential for malignant transformation. Occurs in young, sexually active Bowenoid papulosis males; histologically identical to Bowen disease; presents with multiple reddish brown papules on the glans and is most often transient; virtually never progresses to carcinoma in immunocompetent patients. Appears as a gray, crusted, papular Squamous cell lesion, most commonly on the glans carcinoma of the penis or prepuce, which infiltrates the penis underlying connective tissue to produce an indurated, ulcerated lesion with irregular margins; histologically similar to Bowen disease but with infiltration of the underlying stroma. Scrotum, Testis and Epididymis Represents failure of testicular Cryptorchidism descent into the scrotum, which involves the right testis more commonly than the left; causes increased risk of sterility and development of testicular cancer; most common phase of arrest: inguinoscrotal (4-7th month AOG). Inflammatory disorders most Epididymis commonly affect: testis or epididymis? Clue: Gonorrhea and TB usually affect this organ first. Tumors most commonly involve: Testis testis or epididymis? Clue: Syphilis usually affects this organ first. One of the few true urologic Testicular torsion emergencies, which involves twisting of spermatic cord, which can ultimately lead to hemorrhagic infarction of the testis; golden period for intervention is 6 hours. Most common cause of painless Testicular tumors testicular enlargement. Testicular neoplasms Main difference between biologic Germ cell tumors behavior of testicular germ cell and are usually sex cord-stromal tumors. aggressive; sex cord-stromal tumors are usually benign Most common germ cell tumor; Seminoma (Classic)

Composed of sheets of large, uniform cells with distinct cell borders, clear, glycogen-rich cytoplasm, and round nuclei with conspicuous nucleoli. The cells are often arrayed in small lobules with intervening fibrous septa infiltrated with lymphocytes; 15% have increased hCG due to presence of syncytiotrophoblasts; tumors markers are CD117 (c-kit) and PLAP. Ill-defined invasive masses with foci of necrosis and hemorrhage; Large and primitive-looking with indistinct cell borders, large nuclei, and basophilic cytoplasm; negative for tumor markers. Most common primary testicular neoplasm in children younger than 3 years of age; histologically, low cuboidal to columnar epithelial cells forming microcysts, sheets, glands, and papillae, often associated with eosinophilic hyaline globules; glomeruli-like structures (SchillerDuvall bodies) are present; tumor marker is AFP. A highly aggressive tumor of the trophoblastic lineage; grossly presents as small, nonpalpable masses but with extensive systemic metastases; histologically characterized by presence of cytotrophoblasts (small cuboidal cells) and syncytiotrophoblasts (large, eosninophilic syncytial cells, containing multiple dark pleomorphic nuclei); without villus formation; tumor marker is HCG. Tissues from all three germ-cell layers with varying degrees of differentiation; can contain fully differentiated tissues from one or more germ cell layers (e.g., neural tissue, cartilage, adipose tissue, bone, epithelium) in a haphazard array (mature); or immature somatic elements reminiscent of those in developing fetal tissue (immature). The most important predictor of biologic behavior in testicular teratomas. Non-germ cell malignancy arising in a teratoma; malignant component is chemoresistant; usual cancers include SCCA, mucin-secreting adenoCA, sarcoma). Most common form of testicular neoplasm in men > 60 years of age; most common form is DLBCL; with higher rate of CNS involvement than tumors arising in other areas. Prostate Most common form of prostatitis. Composed of proliferating prostatic glandular elements and fibromuscular stroma that form

Note: Female counterpart is Dysgerminoma

Embryonal carcinoma

Yolk sac tumor/Endodermal sinus tumor/Infantile embryonal carcinoma

Choriocarcinoma

Teratoma

Age (all teratomas in postpubertal males are regarded as malignant) Teratoma with malignant transformation Testicular lymphoma

Chronic abacterial prostatitis Nodular Prostatic Hyperplasia/Benign Prostatic

TOPNOTCH MEDICAL BOARD PREP PATHO SUPERTABLE Page 32 of 47 For inquiries visit www.topnotchboardprep.com.ph or https://www.facebook.com/topnotchmedicalboardprep/

TOPNOTCH MEDICAL BOARD PREP PATHO SUPERTABLE by KEVIN ELOMINA, MD For inquiries visit www.topnotchboardprep.com.ph or https://www.facebook.com/topnotchmedicalboardprep/ nodules through connective tissue septa; the glandular lumina often contain inspissated, proteinaceous secretory material, termed corpora amylacea; stimulus for proliferation is increased circulating androgens (dihydrotestosterone); most commonly occurs is the transition zone. MC cancer in men; most commonly occurs in the peripheral zone; grossly, may appear as firm gray-white lesions with ill-defined margins invading the gland; histologically, may form gland-like structures without basal cell layer, appears crowded, and without branching and papillary infoldings; tumor marker is PSA that is used for adjunct for screening and surveillance. Grading systems used in prostatic adenocarcinoma. Ureter, Bladder and Urethra Most common cause of hydronephrosis in children. Most common and most serious congenital anomaly of the urinary bladder, due to predisposition to infection and scarring. Bladder exstrophy, and urachal anomalies predispose patients into what histologic type of bladder cancer? Most common presentation of bladder cancer. Most common type of bladder cancer; has two premalignant lesions: 1. Noninvasive papillary tumor; and 2. Flat noninvasive carcinoma (CIS); associated with cigarette smoking and occupational carcinogens. Current advancement in the treatment of non-invasive urothelial carcinomas; also a vaccine that is part of EPI, but is administered intravesically. Histologic type of bladder cancer associated with Schistosoma haematobium infection. Most common cause of urinary bladder obstruction in males and females, respectively. Changes that occur in the bladder with obstruction.

Hyperplasia

Prostatic Adenocarcinoma

Gleason and WHO group grade system Ureteropelvic junction obstruction Vesicoureteral reflux Bladder adenocarcinoma Painless hematuria Urothelial (Transitional cell) carcinoma (90%)

Bacille-CalmetteGuerin (BCG) Squamous cell carcinoma Nodular prostatic hyperplasia, cystocele Thickening of wall, Trabeculations, and Diverticula

CHAPTER 21: THE FEMALE GENITAL TRACT QUESTION ANSWER FEMALE GENITAL TRACT

the epidermis, basal cell degeneration, hyperkeratosis, sclerotic changes of superficial dermis, dermal lymphocytic infiltrate; not premalignant, but symptomatic form has increased risk of vulvar cancer. Lesion characterized by acanthosis, hyperkeratosis, dermal lymphocytic infiltrate with evident mitosis; not considered premalignant. Lesions of the anogenital area that may be papillary and distinctly elevated or may be somewhat flat and rugose; characteristic cellular morphology is the presence of cytoplasmic vacuolization with nuclear angular polymorphism and koilocytic atypia: hallmark of HPV infection; not precancerous. HPV subtypes associated with condyloma acuminata. Most common histology of vulvar carcinoma.

Clinically presents as eczematoid lesion on the vulva; characterized by proliferation of malignant epithelial cells within the epidermis; usually not associated with underlying carcinoma, unlike its breast counterpart; PAS-positive (positive for mucin), unlike vulvar melanoma. Vagina Most common histology of vaginal carcinoma. A soft polypoid mass, which is a rare form of primary vaginal cancer; Usually encountered in infants and children less than 5 y/o. Condition characterized by Areas of columnar mucinous epithelium (endocervical-like); 35-90% of patients with in utero exposure to DES; associated with clear cell CA of vagina. Cervix Premalignant lesion of the cervix; can be low-grade (I; <1/3 of the entire epithelium), moderate (II; up to 2/3 of the entire epithelium), and severe (III or CIS; full-thickness of epithelium without invasion of the basement membrane). Another term for CIN I in the recent two-tiered classification system; regresses in 60% of cases.

Squamous hyperplasia Condyloma acuminata

HPV 6 and 11 Squamous cell carcinoma (Note: most vulvar Cas are non-HPV related (70%)) Vulvar Paget disease

Squamous cell carcinoma Sarcoma botryoides (Embryonal rhabdomyosarcoma) Vaginal adenosis

Cervical intraepithelial neoplasia (CIN)

Low-grade squamous Vulva intraepithelial Cystic dilation of the Bartholin gland Bartholin cyst lesion (LSIL) due to duct obstruction; may be Another term for CIN II and III in the High-grade infected and may lead to abscess recent two-tiered classification squamous formation; occurrence in women >40 system; persists in 60% of cases, intraepithelial years old requires biopsy to rule out with 10% of cases progressing to lesion (HSIL) carcinoma. carcinoma within 10 years. Lesion characterized by thinning of Lichen sclerosus TOPNOTCH MEDICAL BOARD PREP PATHO SUPERTABLE Page 33 of 47 For inquiries visit www.topnotchboardprep.com.ph or https://www.facebook.com/topnotchmedicalboardprep/

TOPNOTCH MEDICAL BOARD PREP PATHO SUPERTABLE by KEVIN ELOMINA, MD For inquiries visit www.topnotchboardprep.com.ph or https://www.facebook.com/topnotchmedicalboardprep/ High-risk HPV implicated in the development of cervical carcinoma. Viral oncoprotein responsible for koilocytic atypia in HPV-infected cells. HPV viral gene products that promote tumorigenesis and the tumor suppressor genes they inactivate. Most commonly develops in the transformation zone of the cervix; produces a "barrel cervix" if the tumor encircles the cervix and invades the underlying stroma; most common histology is squamous cell carcinoma (80%), followed by adenocarcinoma (15%) and neuroendocrine carcinoma (5%); all of the types are caused by HPV. Most common cause of death in cervical cancer. Body of Uterus Most common cause of dysfunctional uterine bleeding. Refers to the growth of the basal layer of the endometrium down to the myometrium; nests of endometrial stroma, glands or both are found in the myometrium, in between muscle bundles. Characterized by the presence of endometrial glands and stroma in a location outside the myometrium. Exophytic masses that project into endometrial cavity; may be clinically silent or may cause abnormal uterine bleeding.

Tumor suppressor gene commonly mutated in atypical endometriosis, endometrial hyperplasia and endometrial carcinomas; also mutated in endometrioid and clear cell carcinomas of the ovary. Precursor lesion of endometrial carcinoma secondary to prolonged and marked increased estrogen to progestin ratio; can be simple or complex, with or without atypia. Atypical hyperplasia is now also called endometrial intraepithelial neoplasia (EIN). Most common malignancy of the female genital tract. Most common histology of endometrial carcinoma found in 80% of cases; associated with atypical hyperplasia; maybe exophytic or infiltrative; histologically resembles endometrial glands to solid sheets, depending on differentiation; lowgrade nuclei is a usual feature. Accounts for 15% of cases of endometrial carcinoma; forms small tufts and papillae with high nuclear grade.

HPV 16, 18, 31 and 33 E5 E6 (inactivates p53) E7 (inactivates Rb) Invasive cancer of the cervix

Uremia

Anovulatory cycle Adenomyosis

Endometriosis Endometrial polyp PTEN

Endometrial hyperplasia

Endometrial carcinoma Endometrioid carcinoma (Type I)

Serous carcinoma (Type II)

Common gene mutated in leiomyomas and leiomyosarcomas. Most common benign tumor in females; grossly appears as sharply circumscribed firm gray white masses with a characteristic whorled cut surface; histologically similar to normal smooth muscle cells. Malignant counterpart of leiomyoma; grossly appears as soft, hemorrhagic, necrotic masses; diagnostic features include: tumor necrosis, cytologic atypia and mitotic activity; necrosis being the most important criterion among the three. Ovaries Most common major type of ovarian tumors; 90% of which are malignant. Most common ovarian epithelial tumors; tumors with tubal-like epithelium; most are benign or borderline (70%). Grossly appears as large, spherical structures with smooth glistening serosal covering with less papillary projections; histologically, with a single layer of tall columnar epithelial cells without atypia and invasion; usually with Psammoma bodies. Serous cystadenoma with micropapillary architecture and epithelial stratification, mild nuclear atypia but without invasion of stroma. Grossly appears as large, spherical structures with nodular serosal covering with prominent papillar projections; histologically, with layers of anaplastic epithelial cells and invasion; usually with Psammoma bodies; cancers are divided into low-grade (welldifferentiated) or high-grade (moderately-differentiated or poorly-differentiated), based on general architecture, nuclear atypia and severity of stromal invasion. Mutations associated with low-grade serous carcinomas. Mutation associated with high-grade serous carcinomas. Mucin-containing cystic masses; can be benign or malignant, but most of them are benign; tend to be larger, multicystic and unilateral; can rupture into the peritoneum and produce mucinous ascites (pseudomyxoma peritonei). Most consistent mutation in mucinous tumors. Tumors are usually malignant; 1520% coexist with endometriosis; histologic hallmark is presence of tubular glands resembling benign or malignant endometrium.

MED12 Leiomyoma

Leiomyosarcoma

Surface epithelialstromal tumors (6570%) Serous tumors

Serous cystadenoma

Bordeline serous tumor

Serous cystadenocarcinoma KRAS, BRAF, ERBB2 TP53 Mucinous tumors

KRAS Endometrioid tumors

TOPNOTCH MEDICAL BOARD PREP PATHO SUPERTABLE Page 34 of 47 For inquiries visit www.topnotchboardprep.com.ph or https://www.facebook.com/topnotchmedicalboardprep/

TOPNOTCH MEDICAL BOARD PREP PATHO SUPERTABLE by KEVIN ELOMINA, MD For inquiries visit www.topnotchboardprep.com.ph or https://www.facebook.com/topnotchmedicalboardprep/ Metastatic mucinous adenocarcinoma of the ovary from a gastrointestinal primary; often produces bilateral ovarian masses. Cystic, lined by skin-like structures; usually occur in reproductive age women; histologically, derivatives from more than one germ layer can be seen; generally benign. Solid masses that usually occur in younger women and children; histologically composed of neuroepithelium; generally malignant. A type of teratoma that produces hyperthyroidism due to presence of mature thyroid tissue. Ovarian tumor that may elaborate large amounts of estrogen; two thirds occur in post-menopausal women; potentially malignant; histologically composed of mixture of cuboidal granulosa cells in cords, sheets, or strands and spindled or plump lipid-laden theca cells; granulosa elements may recapitulate ovarian follicle as Call-Exner bodies. Sex cord tumor that recapitulates development of testis with tubules or cords and plump pink Sertoli cells; masculinizing and rarely malignant. Sex cord stromal tumor composed of solid gray fibrous cells to yellow (lipid-laden) plump thecal cells; rarely malignant; associated with hydrothorax and ascites (Meig syndrome). Diseases of Pregnancy A type of hydatidiform mole with the following features: diploid karyotype with diffuse villus edema and trophoblast proliferation; markedly elevated hCG with increased risk of developing subsequent choriocarcinoma. A type of hydatidiform mole with the following features: triploid karyotype with focal villus edema and trophoblast proliferation; less elevated hCG with rare chance of developing subsequent choriocarcinoma. Complete moles that are more invasive locally but do not have the aggressive metastatic potential of a choriocarcinoma. Main difference between ovarian and gestational choriocarcinoma.

Krukenberg tumor

Mature teratoma

Immature teratoma

Struma ovarii Granulosa cell tumor

Sertoli-Leydig cell tumor Thecoma-fibroma

Complete hydatidiform mole

Incomplete (Partial) hydatidiform mole

Invasive mole

Ovarian chorioCAs are more difficult to treat

CHAPTER 22: THE BREAST Non-proliferative breast changes Multifocal, bilateral blue-brown cysts Fibrocystic change ("blue dome cysts") of the breast, measuring 1-5 cm diameter, filled with serous turbid fluid. Occurs normally in the menstrual cycle; histologically, cysts are lined with large and polygonal

with abundant granular, eosinophilic cytoplasm and small, round, deeply chromatic nuclei (apocrine metaplasia); almost always benign. Proliferative breast diseases without atypia Defined as having more than two layers Epithelial of lining epithelium (normally, one hyperplasia layer for luminal, and another for myoepithelial cells). Type of fibrocystic change Sclerosing characterized by proliferation of adenosis luminal spaces (adenosis) lined by epithelial cells and myoepithelial cells with massive stromal fibrosis; important as it appears like carcinoma. Clinically presents with bloody or Intraductal serous discharge; tumors usually are papilloma solitary and less than 1 cm in diameter, consisting of delicate, branching growths within a dilated duct; microscopically, composed of multiple papillae, each having a connective tissue core covered by double layer of epithelial cells (important distinguishing point from carcinoma counterpart). These are stellate lesions characterized Complex by a central nidus of entrapped glands sclerosing in a hyalinized stroma; important as it lesion/Radial scar appears like carcinoma Proliferative breast diseases with atypia Basically, DCIS without full duct Atypical ductal involvement; involves monomorphic hyperplasia (ADH) proliferations of cells (vs. heterogeneous in epithelial hyperplasia). Basically, LCIS but <50% of acini in a Atypical lobular lobule are involved; composed of hyperplasia (ALH) monomorphic, loosely cohesive cells, with absence of E-cadherin. Carcinomas Most common location of breast Upper outer carcinoma. quadrant (50%) The two non-invasive breast Ductal CIS (DCIS) carcinomas; both of which arise from and Lobular CIS terminal duct lobular unit (TDLU). (LCIS) Immunostain to determine presence of p63 myoepithelial layer; an important immunostain to distinguish in situ disease from infiltrating carcinomas. A distinct type of DCIS characterized by Comedocarcinoma highly pleomorphic nuclei of cells (high-grade nuclei) with extensive central necrosis; with calcifications either from calcified necrotic debris or secretory material. Clinically presents as an eczematoid Paget disease of lesion on the breast; associated with an the nipple invasive carcinoma in most cases; cause is an underlying DCIS that extended up to the lactiferous ducts and skin. CIS composed of monomorphic cells LCIS with bland round nuclei (low-grade nuclei) that occur in loosely cohesive clusters within the lobules; tend be more commonly bilateral.

TOPNOTCH MEDICAL BOARD PREP PATHO SUPERTABLE Page 35 of 47 For inquiries visit www.topnotchboardprep.com.ph or https://www.facebook.com/topnotchmedicalboardprep/

TOPNOTCH MEDICAL BOARD PREP PATHO SUPERTABLE by KEVIN ELOMINA, MD For inquiries visit www.topnotchboardprep.com.ph or https://www.facebook.com/topnotchmedicalboardprep/

Most common of the infiltrating carcinomas; associated more with DCIS; characterized with intense desmoplasia that produces a hard palpable mass; microscopically, ranges from well-differentiated (tubule formation) to poorly differentiated (sheets of anaplastic cells); 50-65% are ER(+), HER2 (+) (luminal). The system used in grading invasive carcinomas of no special type.

Infiltrating carcinoma with cells looking like those seen in LCIS; only in single-file conformation (“Indian filing”); reflecting loss of E-cadherin function, a molecule necessary for cohesion of breast epithelial cells; associated with adjacent LCIS in 2/3 of cases. Clinically presents as an enlarged, swollen, erythematous breast with a palpable mass; histologically highgrade; involves dermal lymphatic spaces, which explains the characteristic peau d’orange appearance; poor prognosis. Infiltrating carcinoma composed of sheets large anaplastic cells, with increased mitosis, with wellcircumscribed pushing borders with a pronounced lymphoplasmacytic infliltrate; increased incidence in patients with BRCA1 mutations; usually triple-negative breast cancer; poor prognosis. Stromal tumors The most common benign neoplasm of the female breast; grossly, discrete, usually solitary, freely movable nodule, 1 to 10 cm in diameter, easily "shelled out" lesion of the breast; histologically there is a loose fibroblastic stroma containing duct-like, spaces lined by a layer of epithelium that are regular and have a well-defined, intact basement membrane. Biphasic tumor composed of highly cellular stromal elements forming leaflike projections and epithelial elements; usually benign and localized; ominous changes suggesting malignancy include increased stromal cellularity, anaplasia, high mitotic activity, rapid increase in size, and infiltrative margins. Diseases of the Male Breast Characterized by increase in connective tissue and epithelial hyperplasia of the ducts; lobule formation is rare. Reason why male breast cancer tend to have less favorable prognosis than female breast cancer.

Invasive (ductal) carcinoma (No special type)



Nottingham system (Tubule formation, Nuclear grade, and Mitosis) Invasive lobular carcinoma

Inflammatory carcinoma

Medullary carcinoma

Fibroadenoma

Phyllodes tumor

Gynecomastia Less tissue for the tumor to invade in order to produce metastases

TOPNOTCH MEDICAL BOARD PREP PATHO SUPERTABLE Page 36 of 47 For inquiries visit www.topnotchboardprep.com.ph or https://www.facebook.com/topnotchmedicalboardprep/

TOPNOTCH MEDICAL BOARD PREP PATHO SUPERTABLE by KEVIN ELOMINA, MD For inquiries visit www.topnotchboardprep.com.ph or https://www.facebook.com/topnotchmedicalboardprep/ CHAPTER 23: THE ENDOCRINE SYSTEM component. QUESTION ANSWER Granulomatous inflammation of the Subacute PITUITARY thyroid and viral infections; clinically Granulomatous (De presents as painful thyroiditis. Quervain) Main histologic feature of pituitary Monomorphic cells thyroiditis adenomas. without a significant reticulin network Most common pituitary adenomas. Prolactin cell adenoma; followed by Somatotroph Lymphocytic inflammation of the Subacute (GH) cell adenoma thyroid (morphologically similar to Lymphocytic The main pathology of Cushing Corticotroph cell Hashimoto but without Hurthle cell thyroiditis disease (not syndrome). adenoma change, fibrosis); commonly seen in Associated with mass effects and Nonfunctioning postpartum patients; clinically hypopituitarism (secondary to pituitary adenomas presents as painless thyroiditis. destruction of the normal pituitary Characterized by extensive fibrosis of Reidel thyroiditis parenchyma). the thyroid gland; associated with The only definitive criterion for Metastases primary retroperitoneal fibrosis diagnosis of pituitary carcinoma. (Ormond disease) and autommune Atypical adenomas with metastases; Pituitary carcinoma IgG4-related disease; pathology is usually functional, with ACTH as the unknown, but postulated to be of most common hormone produced, autoimmune type. followed by prolactin. Graves diseases Posterior pituitary syndromes. Diabetes insipidus Triad of Graves disease. Thyrotoxicosis, and SIADH ophthalmopathy The origin of this tumor is from Craniopharyngioma and dermopathy vestigial remnants of Rathke pouch; Main pathology of Graves disease. Presence of thyroid with bimodal age incidence (5-15 stimulating years; 65 years); has immunoglobulin adamantinomatous, and papillary (TSIs); binding to types. TSH receptors THYROID causes activation of Hyperthyroidism and hypothyroidism follicular cell Most common cause of primary Diffuse toxic function hyperthyroidism. hyperplasia Histologic features of Graves disease. Diffuse (Graves disease) hypertrophy and (85%) hyperplasia of the Most common cause of congenital Iodine deficiency follicles with hypothyroidism worldwide. scalloped colloid (moth-eaten) Most common cause of Hashimoto (because of active hypothyroidism in iodine-sufficient thyroiditis reabsorption of areas. follicular cells) Hypothyroidism in infancy and Cretinism Diffuse and multinodular goiter; usually hypothyroid (and childhood, impaired development of compensatory TSH increase leads to proliferation of the CNS (Mental retardation) and skeletal gland) system (short stature), coarse facial Goiter that has 2 phases: 1. Colloid goiter features, protruding tongue, and Hyperplastic phase (due to TSH umbilical hernia. influence), and 2. Colloid phase Hypothyroidism in late childhood and Myexedema (involution of gland due to sufficient adults; and slowing of physical and iodine intake or increased thyroid mental activity; overweight, hormone demand) hypercholesterolemia, nonpitting Repeated hyperplastic and colloid Multinodular goiter edema, coarse facial features, phases results in irregular macroglossia and deepening of voice; involvement of the thyroid gland, accumulation of matrix substances, leading to a condition known as: such as glycosaminoglycans and Multinodular toxic goiter Plummer syndrome hyaluronic acid, in skin, subcutaneous characterized by development of tissue, and a number of visceral sites. autonomously functioning nodules; Thyroiditides presents with hyperthyroidism not Histologically characterized by dense Hashimoto associated with ophthalmopathy and lymphocytic infiltrate with germinal thyroiditis dermopathy. centers; follicles are atrophic with Thyroid neoplasms Hurthle cell change (cells with Clinical characteristics favoring Solitary, young, eosinophilic granular cytoplasm); malignancy in thyroid nodules. male, history of associated with antibodies against radiation therapy, thyroglobulin and thyroid peroxidase; "cold" nodules Type IV hypersensitivity with Type II TOPNOTCH MEDICAL BOARD PREP PATHO SUPERTABLE Page 37 of 47 For inquiries visit www.topnotchboardprep.com.ph or https://www.facebook.com/topnotchmedicalboardprep/

TOPNOTCH MEDICAL BOARD PREP PATHO SUPERTABLE by KEVIN ELOMINA, MD For inquiries visit www.topnotchboardprep.com.ph or https://www.facebook.com/topnotchmedicalboardprep/ Normal looking follicular cells surrounded by an intact capsule; can also be composed of follicular cells that underwent Hurthle cell change (Hurthle cell adenoma); solitary; Note: adenomatous nodule, which is a part of multinodular goiter, is the more appropriate diagnosis for multiple nodules showing adenomatous change. Main distinguishing feature of follicular carcinoma from follicular adenoma.

Follicular adenoma

Capsular and vascular invasion Papillary carcinoma

Most common form of thyroid cancer; histologically characterized by papillary architecture with dense fibrovascular cores; nucleus has finely dispersed chromatin (ground-glass or Orphan Annie nuclei); with Psammoma bodies; spreads by lymphatic route (vs. follicular carcinoma (hematogenous)). Carcinoma arising from parafollicular Medullary Thyroid C cells (secretes calcitonin); Cancer (MTC) histologically characterized by amyloid deposits; known for its multicentricity and association with MEN2 syndromes. Highly lethal thyroid cancer Undifferentiated (mortality rate of 100% in 6 months) carcinoma with a highly pleomorphic (Anaplastic morphology; clinically presents with carcinoma) rapidly enlarging thyroid mass; immunostaining for thyroglobulin is negative. Parathyroid Most common cause of primary Parathyroid hyperparathyroidism; almost always Adenoma (85-95%) confined to one gland (compared to hyperplasia that involves multiple glands); may sometimes show atypia. Definitive criteria for diagnosis of Metastases and parathyroid carcinoma. local invasion Most common cause of secondary Renal failure hyperparathyroidism. Most common cause of Iatrogenic hypoparathyroidism. Endocrine pancreas Diabetes Mellitus (Clinical and physiologic aspects left to clinical subjects) Absolute deficiency of insulin Type I DM secondary of beta cell destruction by an autoimmune process; accounts for only 10% of cases of DM; Type IV hypersensitivity with Type II component; histologically characterized by mononuclear infiltration of the islets with reduction of islet size and number (insulitis). Peripheral resistance to insulin and Type 2 DM relative insulin deficiency is the main

pathology; most common form of DM; histologically characterized by amyloid replacement of islets. The hallmark of diabetic macrovascular disease, which explains its association with coronary artery disease and cerebrovascular disease. Histologic characteristics of diabetic microangiopathy.

Nodular glomerulosclerosis (Kimmelsteil-Wilson lesion); Hyaline arteriolosclerosis; and necrotizing papillitis from pyelonephritis are lesions encountered in: Retinal hemorrhages, microaneurysms, venous dilations, edema, exudates and thickening of retinal capillaries are lesions encountered in what form of diabetic retinopathy? Neovascularization and fibrosis are lesions encountered in what form of diabetic retinopathy?

Pancreatic neuroendocrine tumors Most common PanNET; associated with Whipple’s triad: 1. Hypoglycemia <50 mg/dL; 2. Neuroglycopenic symptoms; and 3. Relief with feeding or parenteral administration of glucose; Biologically favorable behavior; characterized by deposition of amyloid. Associated with Zollinger-Ellison syndrome: 1. Pancreatic islet cell tumor; 2. Hypersecretion of gastric acid; and 3. Peptic ulceration; also associated with MEN-1; biologically aggressive behavior. Adrenal cortex Most common cause of Cushing syndrome (hypercortisolism). Most common cause of endogenous hypercortisolism. Morphologic change in adrenal cortex of patients with ACTH-dependent Cushing syndrome. Morphologic change in adrenal cortex of patients with ACTH-independent Cushing syndrome. (Cortisolsecreting adrenal adenoma). Most common cause of primary hyperaldosteronism. The most common enzymatic defect in congenital adrenal hyperplasia. Most common cause of primary adrenal insufficiency (Addison disease). The only criterion to distinguish adrenocortical carcinoma from an

Accelerated atherosclerosis

Diffuse thickening of the basement membrane, with more leaky capillaries Diabetic nephropathy

Nonproliferative (background)

Proliferative Insulinoma

Gastrinoma

Exogenous administration of steroids ACTH-secreting pituitary adenoma (Cushing disease) Diffuse hyperplasia Cortical atrophy

Bilateral idiopathic hyperplasia (60%) 21-hydroxylase deficiency Autoimmune adrenalitis (6070%) Metastases (adenomas tend to

TOPNOTCH MEDICAL BOARD PREP PATHO SUPERTABLE Page 38 of 47 For inquiries visit www.topnotchboardprep.com.ph or https://www.facebook.com/topnotchmedicalboardprep/

TOPNOTCH MEDICAL BOARD PREP PATHO SUPERTABLE by KEVIN ELOMINA, MD For inquiries visit www.topnotchboardprep.com.ph or https://www.facebook.com/topnotchmedicalboardprep/ adrenocortical adenoma. have atypia as well) Adrenal medulla Tumor of chromaffin cells; Pheochromocytoma histologically composed of polygonal to spindle-shaped cells with finely granular cytoplasm, arranged in nests called zellballen; only criteria for malignancy is metastases; 10% extraadrenal; 10% bilateral; 10% biologically malignant; 10% are not associated with hypertension. Multiple endocrine neoplasia syndromes MEN with 3Ps: Parathyroid (Primary MEN 1 (Wermer hyperparathyroidism either syndrome) secondary to hyperplasia or adenoma), Pancreas (either Gastrinoma or Insulinoma), and Pituitary (most common is Prolactinoma, followed by Somatotroph adenoma); Autosomal dominant; associated with mutations in MEN1 gene on 11q13. MEN with MTC, Pheochromocytoma MEN 2A (Sipple and primary hyperparathyroidism; syndrome) Autosomal dominant; associated with gain of function mutations on RET proto-oncogene in 10q11.2 MEN with MTC, Pheochromocytoma, MEN 2B ganglioneuromas and Marfanoid habitus; associated with different mutations on RET proto-oncogene.

CHAPTER 24: THE SKIN Note: Please familiarize yourselves with the dermatologic and dermatopathologic lexicon before proceeding. QUESTION ANSWER Disorders of pigmentation and melanocytes Disease of depigmentation secondary to Vitilligo destruction of melanocytes. Disease of depigmentation secondary to Albinism decreased or absent synthesis of melanin. These are nevi, usually <6 mm, benignMelanocytic looking, clinically and histologically, and nevus are removed for cosmetic reasons; these lesions have RAS mutations but most never progress because of intact p16 (tumor-suppressor) activity. These are large nevi (>5 mm) and may Dysplastic nevi occur as hundreds of lesions on the body surface; they are flat macules to slightly raised plaques, with a "pebbly" surface; considered as a marker of melanoma risk. The most deadly of all skin cancers; Melanoma results from excessive sun exposure; malignant cells have large nuclei with irregular contours having chromatin characteristically clumped at the periphery of the nuclear membrane and prominent eosinophilic nucleoli often described as "cherry red"; has both radial and vertical growth phases. Immunohistochemical staining for S-100, and HMBmelanoma. 45 (more specific) Benign and premalignant tumors Clinically mimics melanoma; round, Seborrheic exophytic, coin-like plaques varying in keratosis diameter, with a velvety/granular surface; tan to dark brown in color; it has a stuck-on appearance often seen in older individuals; the lesions consist of an orderly proliferation of uniform, monotonous sheets of small cells (basaloid in appearance) with a tendency to form keratin microcysts (horn cysts). This lesion is usually the result of Actinic keratosis chronic exposure to sunlight; dermis contains thickened, blue-gray elastic fibers or "solar elastosis" which is the result of chronic sun damage; grossly, lesions are less than 1cm, tan-brown or red in color, with sandpaper-like surface; microscopically, there is Atypical dyskeratotic cells in basal epidermis with intercellular bridges, solar elastosis, hyperparakeratosis and dermal chronic inflammation. Malignant epithelial tumors A common tumor arising on sunSquamous cell exposed sites in older people, with carcinoma higher incidence in women; may arise from prior actinic keratoses, then when advanced becomes nodular and may ulcerate; characterized by anaplastic (seen on all levels of the epidermis), rounded cells with foci of necrosis and only abortive, single-cell keratinization

TOPNOTCH MEDICAL BOARD PREP PATHO SUPERTABLE Page 39 of 47 For inquiries visit www.topnotchboardprep.com.ph or https://www.facebook.com/topnotchmedicalboardprep/

TOPNOTCH MEDICAL BOARD PREP PATHO SUPERTABLE by KEVIN ELOMINA, MD For inquiries visit www.topnotchboardprep.com.ph or https://www.facebook.com/topnotchmedicalboardprep/ (dyskeratosis). This is the most common human cancer, which is a slow-growing tumor that rarely metastasizes; tends to occur at sites subject to chronic sun exposure and in lightly pigmented people; These tumors present as pearly, smoothsurfaced papules, often containing prominent, dilated subepidermal blood vessels (telangiectasia); the cells have scant cytoplasm, small hyperchromatic nuclei, and a peripheral palisade with clefting from the stroma. Acute inflammatory dermatoses Condition caused by local mast cell degranulation; histologically, there is usually a sparse superficial perivenular infiltrate of mononuclear cells with superficial dermal edema. This term is the accumulation of edema fluid within the epidermis; characterizes all forms of eczamatous dermatitis. An uncommon, usually self-limited disorder that seems to be a hypersensitivity response to certain infections and drugs; patients present with an array of "multiform" lesions, including macules, papules, vesicles, and bullae, as well as the characteristic targetoid lesion consisting of a red macule or papule with a pale vesicular or eroded center; part of a spectrum with SJS-TEN. Chronic inflammatory dermatoses A skin disorder whose main pathology is increased epithelial cell turnover; there is acanthosis and loss of the stratum granulosum with extensive overlying parakeratotic scale; there is also a regular downward elongation of rete ridges (test tubes in a rack appearance); associated with multiple punctate hemorrhages upon removal of scales from the lesions (Auspitz sign) due to presence of dilated tortuous vessels. Small aggregates of neutrophils within the parakeratotic stratum corneum (Munro microabscesses); and spongiotic superficial epidermis (Pustules of Kogoj) are findings associated with: "Pruritic, purple, polygonal, planar papules, and plaques" describes this disorder of the skin and mucosa. Also noted grossly are Wickham striae, which are white lacelike markings over the papules; pattern of inflammation of this disorder is characterized by angulated, zigzag contour ("sawtoothing") of the dermoepidermal junction.

Basal cell carcinoma

Urticaria

Spongiosis Erytherma multiforme

Psoriasis

Psoriasis

Lichen planus

Blistering disorders A rare autoimmune blistering disorder Pemphigus resulting from loss of integrity of normal vulgaris intercellular attachments within the epidermis and mucosal epithelium; caused by a Type II hypersensitivity reaction (mainly IgG) against desmoglein; histologically characterized by acantholysis immediately above the basal layer. Also an autoimmune (IgG-mediated Bullous against bullous pemphigoid antigens) pemphigoid blistering disease against the epidermal basement membrane components (hemidesmosomes); however, it is characterized by subepidermal nonacantholytic blisters. Associated with celiac disease; clinically Dermatitis characterized by pruritic urticarial; herpetiformis histologically characterized by subepidermal blisters that develop from coalescence of vacuolized microabscesses at the dermal papilla tips. Infectious dermatoses A skin infection caused by either S. Impetigo aureus (characterized by formation of bullae) or S. pyogenes (characterized by appearance of honey-colored crusts); histologically characterized by subcorneal neutrophilic infiltration. A benign lesion cause by low-risk strains Verrucae of HPV (2, 4 and 7); characterized by koilocytic atypia (cytoplasmic vacuolization). CHAPTER 25: BONES, JOINTS, AND SOFT TISSUE TUMORS QUESTION ANSWER BONES Congenital disorders Most common skeletal dysplasia; Achondroplasia characterized by disproportionate shortening of the proximal extremities, bowing of the legs, and a lordotic (swaybacked) posture; pathology is mutations on FGFR3; autosomal dominant. Most common lethal form of Thanatophoric dwarfism; also caused by dysplasia mutations in FGF3; usual cause of death is respiratory insufficiency (underveloped chest cavity). This is a group of hereditary Osteogenesis imperfect disorders caused by defective (OI) "brittle bone synthesis of type I collagen; disease" classic finding of a "blue sclerae" is seen in one type of this disorder; has 4 types; Type II is the most lethal; most commonly autosomal dominant. A disease that manifests with Osteopetrosis (Albersdense but structurally unsound Schonberg disease) bone due to mutations that result "Marble bone disease" to impaired osteoclast function;

TOPNOTCH MEDICAL BOARD PREP PATHO SUPERTABLE Page 40 of 47 For inquiries visit www.topnotchboardprep.com.ph or https://www.facebook.com/topnotchmedicalboardprep/

TOPNOTCH MEDICAL BOARD PREP PATHO SUPERTABLE by KEVIN ELOMINA, MD For inquiries visit www.topnotchboardprep.com.ph or https://www.facebook.com/topnotchmedicalboardprep/ also the first disease to be treated with HSC transplantation. Acquired diseases The hallmark of this disease is loss of bone; cortices are thinned and trabeculae are reduced in thickness; osteoclastic activity is present but not dramatically increased; mineral content of the bone tissue is normal. Basic difference between osteopenia and osteoporosis. Characterized by repetitive episodes of frenzied, regional osteoclastic activity and bone resorption, followed by exuberant bone formation, and finally by an apparent exhaustion of cellular activity; pathognomonic feature is a "mosaic pattern" of lamellar bone; associated with secondary osteosarcoma. Condition that results from Vitamin D deficiency; characterized by undermineralization of bone (as opposed to osteoporosis where bone mineral content is normal), rendering one susceptible to fractures. Basic difference between rickets and osteomalacia, and osteoporosis.

Components of von Recklinghausen disease of bone. Term used for skeletal changes in chronic renal failure (including dialysis). Osteomyelitis Most common route of infection of pyogenic osteomyelitis. Most common organism implicated in pyogenic osteomyelitis overall. Common organisms implicated in pyogenic osteomyelitis in IV drug users and patients with GUT infections. Common organisms implicated in pyogenic osteomyelitis in neonates. Organism implicated in osteomyelitis in patients with sickle cell anemia. A clinically serious form of

Osteoporosis

Severe osteopenia enough to increase risk of fractures in osteoporosis Paget disease of bone (Osteitis deformans)

Rickets (children)/ Osteomalacia (adults)

Osteoporosis: normal mineralization, decreased bone mass Rickets/Osteomalacia: decreased mineral content of bone Increased bone cell activity, peritrabecular fibrosis, cystic brown tumors Renal osteodystrophy

Hematogenous dissemination S. aureus E. coli, Pseudomonas, Klebsiella H. influenzae and GBS (organisms involves in sepsis) Salmonella Pott disease

tuberculous osteomyelitis involving vertebral bodies; can cause vertebral deformity, collapse and posterior displacement leading to neurologic deficits. Tumors Bone-forming tumors Bone-forming tumors with marked reactive bone (nidus) formation; clinically, <2 cm lesions; painful but relieved by NSAIDs. Basic difference between osteoid osteoma/osteoblastoma and fibrous dysplasia. A bone-producing malignant mesenchymal tumor, characterized as gritty, graywhite tumors, often exhibiting hemorrhage and cystic degeneration; the production of mineralized or unmineralized bone (osteoid) by malignant cells is essential for diagnosis; associated with Codman triangle and sunburst pattern, radiographically; metaphyseal. Most common benign bone tumor, characterized by hyaline cartilage-capped outgrowths attached by a bony stalk to the underlying skeleton; metaphyseal. Second most common malignant matrix=producing tumor of bone; a malignant tumor of chrondrocytes; these lesion arises within the medullary cavity of the bone to form an expansile glistening mass that often erodes the cortex; they exhibit malignant hyaline and myxoid cartilage; in its conventional form, it is almost always never found in children. Second most common group of bone sarcomas in children; primary malignant small roundcell tumor of bone and soft tissue, characterized by sheets of small round cells with scant, cleared cytoplasm, circled about a central fibrillary space or "HomerWright rosettes"; basically a PNET of bone; associated with a chromosomal translocation abnormality (t(11:22)). A relatively uncommon benign tumor usually arising in individuals in their 20s to 40s; these are large and red-brown

Osteoid osteoma Note: osteoblastomas are > 2 cm, and pain is not relieved by NSAIDs; morphologically, less nidus formation Presence of osteoblastic rimming in osteoid osteoma/osteoblastoma Osteosarcoma

Osteochondroma

Chondrosarcoma (Note: If you see malignant tumor made of chrondrocytes in a child, think chondroblastic osteosarcoma first)

Ewing sarcoma

Giant cell tumor of bone (Osteoclastoma)

TOPNOTCH MEDICAL BOARD PREP PATHO SUPERTABLE Page 41 of 47 For inquiries visit www.topnotchboardprep.com.ph or https://www.facebook.com/topnotchmedicalboardprep/

TOPNOTCH MEDICAL BOARD PREP PATHO SUPERTABLE by KEVIN ELOMINA, MD For inquiries visit www.topnotchboardprep.com.ph or https://www.facebook.com/topnotchmedicalboardprep/ lesions with frequent cystic degeneration, composed of uniform oval mononuclear cells with frequent mitoses, with scattered osteoclast-type giant cells containing 100 or more nuclei; locally invasive; epiphyseal. Most common skeletal malignancy. JOINTS Most common form of arthritis; most fundamental feature is degeneration of articular cartilage; characterized by fibrillation and cracking of the articular cartilage matrix, bone eburnation, and bony outgrowths/spurs (osteophytes); full-thickness portions of the cartilage are lost, and the subchondral bone plate is exposed. Chronic inflammatory autoimmune disease principally attacking the joints; mechanism of disease is Type IV hypersensitivity; radiographic hallmark of this joint disease are joint effusions and juxta-articular osteopenia with erosion and narrowing of joint space and loss of articular cartilage. Basic difference between OA and RA. Classic lesion of RA composed of proliferating synovial lining cells admixed with inflammatory cells, granulation tissue, and fibrous connective tissue; the overgrowth of this tissue is so exuberant that the usually thin, smooth synovial membrane is transformed into lush, edematous, frondlike (villous) projections. Examples of seronegative spondyloarthropathies.

found in the cytoplasm of the neutrophils as well as in small clusters in the synovium. Pathognomonic feature of gout; Tophi formed by large aggregations of urate crystals surrounded by an intense inflammatory reaction of lymphocytes, macrophages, and Metastases foreign-body giant cells, attempting to engulf the masses of crystals. Crystal-induced arthropathy due Pseudogout Osteoarthritis to calcium pyrophosphate deposition; crystals are rhomboid, and positivelybirefringent. SOFT TISSUE Most common soft tissue tumor Lipoma of adulthood. Most common sarcoma of Liposarcoma adulthood. Most common soft tissue Rhabdomyosarcoma sarcoma of childhood and Rheumatoid arthritis adolescence. (RA) Immunostain for Myogenin rhabdomyoblastic differentiation. Most common neoplasm in Leiomyoma women: common site is the uterus. Immunostains for smooth muscle Smooth muscle actin, differentiation. desmin CHAPTER 26: PERIPHERAL NERVES AND SKELETAL Joint fusion is absent in MUSCLES OA QUESTION ANSWER Pannus Disorders of peripheral nerve Most common chronic acquired Chronic inflammatory peripheral neuropathy; clinical demyelinating feature is mixed sensorimotor polyradiculoneuropathy polyneuropathy ≥ 2 months. (CIDP) Most common inherited Charcot-Marie Tooth peripheral neuropathy; has (CMT) disease autosomal dominant (CMT1 and 2) and CMTX (X-linked) forms; CMT2 is an axonal neuropathy that is phenotypically severe and presents early in life. Rapidly progressive acute Guillain-Barre demyelinating disorder affecting syndrome motor axons that results in Ankylosing spondylitis ascending weakness that may (Rheumatoid lead to death from failure of spondylitis, Marierespiratory muscles over a Strumpell disease), period of only several days; Reactive arthritis associated with C. jejuni infection. (Reiter syndrome), The most common form of Distal symmetric Enteritis-associated diabetic neuropathy (DM sensorimotor arthritis, Psoriatic neuropathy: most common cause neuropathy arthritis of peripheral neuropathy) Crystal-induced arthropathy Crystal-induced arthropathy due Gout to excessive amounts of uric acid; Disorders of neuromuscular junction histologically characterized by a Disorder caused by Myasthenia gravis dense neutrophilic infiltrate autoantibodies that block the permeating the synovium and function of postsynaptic synovial fluid; long, slender, acetylcholine receptors at motor needle-shaped monosodium end plates, which results in the urate crystals are frequently degradation and depletion of the TOPNOTCH MEDICAL BOARD PREP PATHO SUPERTABLE Page 42 of 47 For inquiries visit www.topnotchboardprep.com.ph or https://www.facebook.com/topnotchmedicalboardprep/

TOPNOTCH MEDICAL BOARD PREP PATHO SUPERTABLE by KEVIN ELOMINA, MD For inquiries visit www.topnotchboardprep.com.ph or https://www.facebook.com/topnotchmedicalboardprep/ receptors; clinically presents with fatigable weakness. Disorder caused by Lambert-Eaton autoantibodies that inhibit the myasthenic syndrome function of presynaptic calcium (LEMS) channels, which reduces the release of acetylcholine into the synaptic cleft; associated with SCLC as paraneoplastic syndrome. Acquired disorders of skeletal muscle Most common in inflammatory Dermatomyositis myopathy in children; associated with Gottron papules and heliotrope rash (see Pediatrics); microscopically, it is associated with perivascular mononuclear cell infiltrates, “dropout” of capillaries, the presence of socalled tubuloreticular inclusions in endothelial cells, and myofiber damage in a paraseptal or perifascicular pattern. Basic difference between Skin changes absent in dermatomyositis and polymyositis polymyositis. Most common inflammatory Inclusion body myositis myopathy in patients older than 65 years; clinically present with quadriceps and distal upper extremity weakness, with dysphagia. Muscular dystrophies Disorders caused by mutation in Duchenne Muscular dystrophin gene in X Dystrophy chromosome (Xp21); patients (DMD)/Becker present with weakness at around Muscular Dystrophy 5 years of age; (BMD) (less severe) pseudohypertrophy of the calf muscles is an important initial physical finding; associated with Gower sign. Differences in dystrophin DMD: Dystrophin(-) immunohistochemistry between BMD: Dystrophin(+) but DMD and BMD. reduced Peripheral nerve sheath tumors Mixture of cellular areas (Antoni Schwannoma/ A) and hypocellular areas in Neurilemmoma myxoid stroma (Antoni B) with palisading of nuclei and Verocay bodies; Immunohistochemical staining for S-100 is positive; associated with NF type 2. Tumor composed of Bland Neurofibroma Schwann cells with stromal cells (mast cells, perineurial cells, CD34+ spindle cells and fibroblasts) in a loose collagen stroma; associated with NF type 1. Most common origin of Malignant malignant peripheral nerve transformation of a sheath tumors. plexiform neurofibroma

Neurofibromatosis Type 1

Autosomal dominant disorder caused by mutations in the tumor suppressor neurofibromin a tumor suppressor of Ras oncoprotein, encoded on Chromosome 17 (17q11.2); associated with development of neurofibromas, MPNSTs, optic nerve gliomas, other glial tumors and hamartomatous lesions, pheochromocytomas, pigmented iris nodules (Lisch nodules), and cutaneous hyperpigmented macules (axillary freckling and café au lait spots). Autosomal dominant disorder Neurofibromatosis Type caused by mutations in the 2 merlin gene (NF2), a tumor suppressor that facilitates Ecadherin contact inhibition, in Chromosome 22 (Ch22q12); hallmark is bilateral vestibular schwannomas, meningiomas and ependymomas (commonly intraspinal). CHAPTER 27: THE CENTRAL NERVOUS SYSTEM Note: Congenital anomalies are omitted because of low-yield; for completion, you may refer to the parent handout. Also, some common CNS anomalies are discussed in anatomy (under embryology). CNS infections are covered in Microbiology, and lumbar tap findings for meningitides are covered in IM and Pediatrics. QUESTION ANSWER Cellular pathology of the CNS General response of neurons to injury. Acute: Intense eosinophilia (red neuron) Chronic: cell loss General response of astrocytes and Astrocytes: microglia to injury. Hyperplasia, hypertrophy; with accumulation of GFAP Microglia: proliferation Edema, herniation and hydrocephalus (focus on general concepts) It is the accumulation of excess fluid Cerebral edema within the brain parenchyma. The brain is softer than normal and often appears to "overfill" the cranial vault. In generalized edema the gyri are flattened, the intervening sulci are narrowed, and the ventricular cavities are compressed; can be vasogenic (occurs when the integrity of the normal blood-brain barrier is disrupted; with increased vascular permeability, fluid shifts from the vascular compartment into the intercellular spaces of the brain), or cytotoxic (due to an increase in intracellular fluid secondary to neuronal, glial, or endothelial cell

TOPNOTCH MEDICAL BOARD PREP PATHO SUPERTABLE Page 43 of 47 For inquiries visit www.topnotchboardprep.com.ph or https://www.facebook.com/topnotchmedicalboardprep/

TOPNOTCH MEDICAL BOARD PREP PATHO SUPERTABLE by KEVIN ELOMINA, MD For inquiries visit www.topnotchboardprep.com.ph or https://www.facebook.com/topnotchmedicalboardprep/ membrane injury, as might be encountered in an individual with a generalized hypoxic/ischemic insult or with exposure to some toxins). Refers to the accumulation of CSF Hydrocephalus leading to dilation of the ventricular system; can be noncommunicating (from an obstacle or disruption of flow seen most commonly at the foramen of Monroe or aqueduct of Sylvius), or communicating (due to reduced resorption of CSF; all of the ventricular system is enlarged). Effect of increased intracranial Herniation pressure from an increase in volume of any intracranial component; may lead to vascular compromise, infarction, additional swelling and herniation; has several types including, subfalcine, transtentorial, and tonsillar. CNS trauma (focus on traumatic vascular injuries) This leads to accumulation of blood Epidural between the dura and the skull; the hematoma expanding hematoma has a smooth inner contour that compresses the brain surface; clinically, patients may experience a lucid interval between the moment of trauma and development of neurologic symptoms; bleeding source usually arterial (from middle meningeal artery in pterion fractures); on imaging, typically shows a lentiform bleed. This leads to accumulation of blood in Subdural the subdural space; bleeding is usually hematoma venous (from injury to bridging veins); usually observed in extremes of ages; can be chronic; on imaging, typically shows a crescent-shaped bleed. Cerebrovascular diseases In the setting of this condition, the Global cerebral brain is swollen, with wide gyri and ischemia narrowed sulci; the cut surface shows poor demarcation between gray and white matter; results from generalized reduction of cerebral perfusion, usually below systolic pressures of less than 50mmHg. Neurons most sensitive to ischemia. Pyramidal layer of hippocampus (CA1: Sommer sector), cerebellar Purkinje cells, pyramidal cells of cerebral cortex Results from cerebral artery occlusion, Focal cerebral either thrombotic or embolic; and ischemia hemorrhagic or nonhemorrhagic; nonhemorrhagic infarcts show

liquefactive necrosis of the brain parenchyma. Injury to small perforating vessels in the form of sclerosis due to hypertension; morphologically, small, cavitary infarcts (lacunes). Injury to small perforating vessels, in the form of rupture of small vessels due to hypertension; morphologically, tissue destruction, pigment-laden macrophages and gliosis. Bleeding secondary to rupture of small intraparenchymal vessel; nontraumatic; most common cause of deep parenchymal hemorrhage is hypertension; commonly affects the basal ganglia (putamen), thalamus, and pons. Intraparenchymal hemorrhage affecting the leptomeninges and cortex; due to deposition of amyloid in small to medium-sized leptomingeal and cortical vessels. Patients with this type of intracranial hemorrhage complains of having "the worst headache I've ever had"/ thunderclap headache; the most frequent cause is rupture of saccular berry aneurysm; most common location is ACA-ACoA junction (40%). A vascular malformation wherein the involved vessels resemble a tangled network of wormlike vascular channels; microscopically, they are enlarged blood vessels separated by gliotic tissue, often with evidence of prior hemorrhage; common location vessels involved are of the subarachnoid space and brain. Demyelinating disorders Most common demyelinating disorder; characterized by distinct episodes of neurologic deficits, separated in time, attributable to white matter lesions that are separated in space; affected areas show multiple, wellcircumscribed, slightly depressed, glassy, gray-tan, irregularly shaped lesions, termed "plaques"; common initial manifestation is unilateral involvement of the optic nerve (optic or retrobulbar neuritis). CSF findings in Multiple sclerosis.

Clinically present with bilateral optic neuritis and spinal cord demyelination; associated with antibodies against aquaporin-4. Demyelinating disorder associated with rapid correction of hyponatremia.

Neurodegenerative diseases It is the most common cause of

Lacunar infarcts

Slit hemorrhages

Hypertensive intraparenchymal hemorrhage

Cerebral amyloid angiopathyassociated intraparenchymal hemorrhage Subarachnoid hemorrhage

Arteriovenous malformation

Multiple sclerosis

Moderate pleocytosis, mildly elevated protein, increased IgG (oligoclonal) Neuromyelitis optica (NMO) Central pontine myelinolysis (CPM)/Osmotic demyelination syndrome (ODS) Alzheimer disease

TOPNOTCH MEDICAL BOARD PREP PATHO SUPERTABLE Page 44 of 47 For inquiries visit www.topnotchboardprep.com.ph or https://www.facebook.com/topnotchmedicalboardprep/

TOPNOTCH MEDICAL BOARD PREP PATHO SUPERTABLE by KEVIN ELOMINA, MD For inquiries visit www.topnotchboardprep.com.ph or https://www.facebook.com/topnotchmedicalboardprep/ dementia in the older adults, (AD) characterized by presence of plaques composed of Aβ amyloid, and neurofibrillary tangles composed of Tau proteins. Loss of dopaminergic neurons in Parkinson disease substantia nigra; triad of tremor, (PD) rigidity and bradykinesia; there are single or multiple, intracytoplasmic, eosinophilic, round to elongated inclusions that often have a dense core surrounded by a pale halo or Lewy bodies, composed of α-synuclein aggreagates. Accumulation of Lewy bodies in basal Lewy body nucleus of Meynert leads to this dementia condition: This is the most common form of Amytropic lateral neurodegeneration affecting the motor sclerosis (ALS) system; characterized by muscle atrophy and hypereflexia due to loss of both upper and lower motor neurons. Acquired metabolic and toxic disturbances Psychotic symptoms or Wernicke ophthalmoplegia on a background of syndrome acute alcoholism; reversible with thiamine administration; morphologically, hemorrhage and necrosis of the mamillary bodies and the walls of the third and fourth ventricles Short-term memory disturbances and Korsakoff confabulation on a background of syndrome chronic alcoholism; not reversible with thiamine administration; morphologically, cystic space with hemosiderin-laden macrophages in dorsomedial nucleus of the thalamus. Degeneration of posterolateral cord Subacute tracts due to abnormal myelin combined formation associated with Vitamin B12 degeneration deficiency; clinically, sensory disturbance and ataxia in LEs that leads to spastic paralysis and paraplegia. Tumors Gliomas Cytogenetic origin is astrocytes; forms Astrocytoma include: Pilocytic (WHO I/IV) (common in children), Welldifferentiated (WHO II/IV), Anaplastic (WHO III/IV) and glioblastoma (WHO IV/IV) (Note: the rest except pilocytic astrocytoma are called infiltrating astrocytomas, and are common in adults). Histologic criteria for the diagnosis of Necrosis and glioblastoma. endothelial proliferation Cytogenetic origin is oligodendrocytes; Oligodendroglioma forms include: Well-differentiated (WHO II/IV), and Anaplastic (WHO III/IV); histologically characterized by round tumor cells with cytoplasmic halos ("fried-egg appearance) supplied a by network of anastomosing

capillaries with calcifications. Common location of this tumor is near 4th ventricle during the first two decades of life, and intraspinal in adults; cytogenetic origin is ependymal cells; tend perivascular pseudorosettes on histologic examination. Poorly-differentiated tumors Often located midline in the cerebellum in children, and lateral in adults; on histology, lesion is extremely cellular with sheets of anaplastic (small blue) cells; can also form Homer-Wright rosettes; essentially a primitive neuroectodermal tumor (PNET); radiosensitive. Other tumors Most common form of primary CNS lymphoma. Cytogenetic origin is arachnoid meningothelial cells; varied histologic patterns include syncytial, fibroblastic, transitional, psammomatous and secretory (associated with pseudopsammoma bodies); grades include: Typical (WHO I/IV), Atypical (WHO II/IV); and Anaplastic (Malignant) (WHO III/IV). Sharply demarcated masses, often at the junction of gray and white matter, usually surrounded by a zone of edema; usual sources are lung, breast, skin (melanoma), kidney, and GIT, and choriocarcinomas.

Ependymoma

Medulloblastoma

DLBCL Meningioma

Metastases

TOPNOTCH MEDICAL BOARD PREP PATHO SUPERTABLE Page 45 of 47 For inquiries visit www.topnotchboardprep.com.ph or https://www.facebook.com/topnotchmedicalboardprep/

TOPNOTCH MEDICAL BOARD PREP PATHO SUPERTABLE by KEVIN ELOMINA, MD For inquiries visit www.topnotchboardprep.com.ph or https://www.facebook.com/topnotchmedicalboardprep/ CHAPTER 28: THE EYE QUESTION Orbit Forward displacement of the eye; has two forms: axial and positional. Most common primary tumor of the orbit.

Eyelid Most common malignancy of the eyelid. Conjunctiva Etiologic agent of conjunctivitis that lead to significant conjunctival scarring. Most common site of conjunctival neoplasms. Common histology of common malignancies. Anterior segment Inflammation within the vitreous humor. Inflammation involving the retina, uvea, sclera and orbit. Opacification of the lens. Submucosal fibrovascular connective tissue that does not invade the cornea. Submucosal fibrovascular connective tissue that invades the cornea; associated with visual problems (astigmatism). Uvea Inflammation of the tissues that comprise the uvea (iris, ciliary body and choroid). Most common intraocular malignancy. Most common primary intraocular malignancy in adults. Retina and vitreous Unclean vitreous detachment tears the retina; liquefied vitreous accumulates between RPE and neurosensory retina; involves full-thickness retinal defect. Any condition that damages the RPE and permits fluid to leak from the choroidal circulation under the retina (choroidal tumors, malignant hypertension) leads to this type of retinal detachment; does not involve full-thickness retinal defect. Ophthalmologic emergency; usually occurs with ischemia; histologically characterized by fragments of atherosclerotic plaque lodged in retinal circulation (Hollenhorst plaques); and cherry-red spot in macula (fovea and foveola) with retinal pallor. Retinal hemorrhages, dilated tortuous retinal veins, cotton-wool spots, macular edema, and optic disc edema, are

ANSWER

Proptosis Vascular tumors (Infancy and early childhood: capillary hemangioma, lymphangioma; adults: cavernous hemangioma) Basal cell carcinoma Chlamydia trachomatis Limbus Squamous cell carcinomas, Melanomas Endophthalmitis Panophthalmitis Cataract Pinguecula Pterygium

Uveitis

Metastases to uvea Melanoma Rhegmatogenous retinal detachment Nonrhegmatogenous

Central retinal artery occlusion (CRAO)

Central retinal vein occlusion (CRVO)

findings typical of this condition, which may occur in the presence or absence of ischemia. Most common type of retinal lymphoma.

Most common primary intraocular malignancy in children; clinical findings include poor vision, strabismus, whitish hue when eyes are illuminated (luekocoria); histologically, composed of small, round cells with large hyperchromatic nuclei and scant cytoplasm, with characteristic structures consisting of clusters of cuboidal or short columnar cells arranged around a central lumen; nuclei are displaced away from the lumen, which appears to have a limiting membrane; associated with FlexnerWintersteiner rosettes and fleurettes. Optic nerve Swelling of the optic nerve head; can be unilateral or bilateral (papilledema); unilateral usually caused by nerve compression; bilateral usually caused by increased ICP. Loss of vision secondary to demyelination of the optic nerve; most important cause is multiple sclerosis. Most common primary optic nerve tumors.

DLBCL Retinoblastoma

Optic nerve edema

Optic neuritis Pilocytic astrocytomas and meningiomas

References: • Kumar V, Abbas AK, Aster JC. 2013. Robbins Basic Pathology, 9th ed. Elsevier Saunders PA USA. • Kumar V, Abbas AK, Fausto N, Aster JC. 2010. Robbins and Cotran Pathologic Basis of Disease. 9th ed. Elsevier Saunders PA USA. • Schneider AS, Szanto PA. 2014. BRS Pathology. 5th ed. Lippincott Williams & Wilkins PA USA. • Topnotch Pathology handout • Topnotch Pathology Flashcards ------- There verses are included in my guiding verses in life, and during the boards. It speaks of the power of persistence, and the providence of God to his children. Luke 11:5-13 New King James Version (NKJV) A Friend Comes at Midnight And He said to them, “Which of you shall have a friend, and go to him at midnight and say to him, ‘Friend, lend me three loaves; for a friend of mine has come to me on his journey, and I have nothing to set before him’; and he will answer from within and say, ‘Do not trouble me; the door is now shut, and my children are with me in bed; I cannot rise and give to you’? I say to you, though he will not rise and give to him because he is his friend, yet because of his persistence he will rise and give him as many as he needs. Keep Asking, Seeking, Knocking

TOPNOTCH MEDICAL BOARD PREP PATHO SUPERTABLE Page 46 of 47 For inquiries visit www.topnotchboardprep.com.ph or https://www.facebook.com/topnotchmedicalboardprep/

TOPNOTCH MEDICAL BOARD PREP PATHO SUPERTABLE by KEVIN ELOMINA, MD For inquiries visit www.topnotchboardprep.com.ph or https://www.facebook.com/topnotchmedicalboardprep/ “So I say to you, ask, and it will be given to you; seek, and you will find; knock, and it will be opened to you. For everyone who asks receives, and he who seeks finds, and to him who knocks it will be opened. If a son asks for bread[a] from any father among you, will he give him a stone? Or if he asks for a fish, will he give him a serpent instead of a fish? Or if he asks for an egg, will he offer him a scorpion? If you then, being evil, know how to give good gifts to your children, how much more will your heavenly Father give the Holy Spirit to those who ask Him!"

TOPNOTCH MEDICAL BOARD PREP PATHO SUPERTABLE Page 47 of 47 For inquiries visit www.topnotchboardprep.com.ph or https://www.facebook.com/topnotchmedicalboardprep/

Related Documents

Virus
February 2021 8
Virus
March 2021 0
Virus
February 2021 8
Corona Virus
January 2021 1
Corona Virus
January 2021 1

More Documents from "nur khikmah"